You are on page 1of 28

No MCQ Answers Correct Answer Explanation Source

1 A 78 year old caucasian man was admitted to the ER, with sudden pain in the A. Ultrasonography B. US can be used to detect the presence of an AAA, but it has a low sensitivity to detect a retroperitoneal https://www.esvs.org/wp-
abdomen and back. The pain is described as ‚tearing‘. On admission his HR is hemorrhage. CTA is the gold standard for all patients with suspected rAAA. It allows to image the patients content/uploads/2018/12/Wanhainen-A-et-al-ESVS-AAA-
GL-2019-epublished-041218.pdf
140/min, his BP 68/39 mmHg and his RF is 22/min. GCS is 15, ECG shows a sinus vascular anatomy and is crucial in determination whether EVAR or open surgery is required.
tachycardia without changes in the ST-segment. Abdominal examination reveals a
pulsating mass. Doctors suspect a symptomatic or ruptured AAA and decide to
operate, using EVAR as the method of choice. Considering the findings, which imaging
method is inevitable beforehand?
B. CTA
C. MRI
D. Xray
E. Catheter Angiography

2 A 73 year old man is at the GP for check-up. Incidentally the GP discovered a AAA with A. regular US to follow the course of the aneurysm over A. With an aneurysm <5cm and no symptoms we recommend regular US monitoring, for aneurysms 4.0-4.9cm https://www.esvs.org/wp-
a diameter of 4,5cm. The patient certifies he was unaware of this condition and never time annual surveillance is safe,drugs to reduce AAA growth rate & cardiovascular risk. Surgical treatment is needed content/uploads/2018/12/Wanhainen-A-et-al-ESVS-AAA-
GL-2019-epublished-041218.pdf
experienced pain in the back or abdomen. He did not yet suffer from thromboembolic if the aneurysm is >5,5cm or increasing by more than 0,5cm in 6 month. Emergency surgery - in case of
events. His extremities do not show signs of peripheral artery diseases. He has symptomatic or ruptured AAA.
hypertension, but it is under control with medications. He smokes occasionally and
his cholesterol levels are high. What would you recommend the patient as the next
step?
B. emergency surgery
C. EVAR
D. elective open surgery
E. aneurysmal wrapping

3 A 59 year old man visits the physician for his regular ultrasonic follow-up of his AAA. A. continue to minimize risk factors and come again for D. A growth of 0,5cm in 6 month is an indicator for elective surgical intervention. https://www.esvs.org/wp-
The last years he did not observe any symptoms. He was also really compliant in follow-up in 6 month content/uploads/2018/12/Wanhainen-A-et-al-ESVS-AAA-
GL-2019-epublished-041218.pdf
minimizing his risk factors and over-all he feels good. The diameter was at a stable
4cm the last years (last measured - 6 months ago), this time we observe a diameter of
5cm in US. What would you recommend to your patient as his surgeon?
B. emergency open repair surgery
C. emergency EVAR surgery
D. elective surgical correction
E. nothing, as long as the patient does not show symptoms

4 A 50 yo man comes to the physician because of increasing difficulty achieving an A. Decreased lower leg pulses A Decreased pulses are a common finding in peripheral arterial disease, for which this patient is at high risk. A https://www.mayoclinic.org/diseases-conditions/peripheral-
erection for 6 months. During this period, he has had to reduce his hours at work lack of bilateral pulses sugests occlusion. artery-disease/symptoms-causes/syc-20350557
because of lower back, thigh, as well as lower limb weakness when walking for longer
distances. His pain resolved after resting for few minutes or standing still. His father
has coronary artery disease and his mother died of a ruptured brain aneurysm. He has
smoked a pack of cigarettes daily for 30 years. Last visit to the doctor was 25 years
ago. Physical examination :BMI = 35 BP = 167/98 HB A1c = 6.2% Which additional
finding will be present on physical examination of this patient ?
B. Jugular venous distention
C. Fever and rush on palms and soles
D. Painless ulcer on malleolus
E. Enlarged liver and spleen

5 A 65 yo man with 6 month history of bilateral calf pain. The pain usually comes after A. Propranolol therapy C Vascular bypass surgery or endovascular treatment is the definitive treatment option in patients presenting with https://www.nhlbi.nih.gov/health-topics/peripheral-artery-
walking his dog a few blocks and it's more severe on the right side. He has CAD, rest pain and ulceration. Primary form of treatment should involve minor and major risk factor modification. disease
hypertension, DM 2 and he is smoking 2 packs of cigarettes daily for 40 years. His
medications include metformin, Lisinopril and aspirin. Physical examination: BMI 32
BP : 130 / 72 The legs have shiny skin and reduced hair below the knee. Decrease
lower leg pulses in both legs. Ankle brachial index (ABI) = 0.5 What may a vascular
surgeon suggest as treatment in this case?
B. Spinal cord stimulation
C. Cessation of smoking
D. Tight leg stockings
E. Laser ablation

6 You’re assessing a patient’s health history for peripheral vascular disease. What signs A. “I often feel pain in my legs when I sleep which resolves D Claudication following physical activity is the most common symptom in patients suffering from chronic lower https://www.medicalnewstoday.com/articles/188939#diagno
and symptoms reported by the patient would indicate the patient may be experiencing by moving them constantly .” limb ischemia. sis
peripheral arterial disease?
B. “If I stand or sit too long my legs start to feel heavy and
achy.”
C. “It hurts to elevate my legs.”
D. “Sometimes when I’m walking my legs start to cramp and
tingle
E. to the point
“I usually goes where I can’t walk
out of breath whenuntil the 2pain
I climb goes
flights of
stairs, and have to take a break “
7 A 59-year old woman comes to the physician because of intermittent lightheadedness A. Adverse effect of medications B. Patient most likely presents with subclavian steal syndrome. High-grade stenosis of the subclavian artery, https://www.uptodate.com/contents/subclavian-steal-
and paresthesia of her right arm for the past 3 months. During these episodes she has leads to a decrease in the pressure distal to the lesion. Blood flow in the ipsilateral vertebral artery reverses, syndrome
blurry vision and dizziness. She has hyperlipidemia and type 2 diabetes mellitus for stealing the blood from the unaffected subclavian artery via the contralateral vertebral artery in order to ensure
which she takes atorvastatin, metformin and glipizide. She also says that sometimes adequate blood supply to the involved arm.
her right hand feels colder than her left hand. Physical examination shows decreased
radial pulse on the right upper extremity. Which of the following is the most likely
underlying cause of this patient’s symptoms?
B. Reversed blood flow in the right vertebral artery
C. Compression of neurovascular structure in the neck
D. Infarction of middle cerebral artery
E. Brachiocephalic syndrome

8 A 56-year old man complains about cramping pain in his left arm that is accompanied A. CT angiography C. Patient most likely presents with subclavian steal syndrome. Vigorous exercise of the arm provokes symptoms, https://www.uptodate.com/contents/subclavian-steal-
with a severe headache and nausea every time he is in the gym. During the last time such as paroxysmal vertigo, drop attacks and/or arm claudication. Doppler USG is a useful screening tool as it syndrome
he went to the gym which was yesterday, he fainted which prompted his arrival to your is noninvasive, readily available and cheap, but must be confirmed with MRA\CTA if surgical treatment is
office. He had hypertension for 13 years but refuses to take medications because he indicated.
feels fine. His latest blood test result from 5 years ago showed hyperlipidemia and
HbA1c=8.3. Examination reveals a left subclavian murmur. Which of the following is a
useful screening tool for his condition?
B. MR angiography
C. Doppler USG
D. CXR
E. EKG

9 An Asian 30-year-old woman arrives to the E.R following an episode of syncope. She A. Chronic liver disease D. In Asia, 36% of cases of subclavian steal syndrome are caused by Takayasu's arteritis. These tend to present https://patient.info/doctor/subclavian-steal-phenomenon
is anxious and afraid. A nurse who checked her blood pressure says that there is a 30- at an earlier age (<30 years) and have a female predominance.
mmHg difference between her left and right arm. The woman also reports pain and
paresthesia in her left arm, as well as dizziness and blurry vision from time to time.
Her left radial pulse is weak and delayed. A doppler ultrasound was done, and a CT
angiography confirmed her diagnosis. Which of the following conditions is associated
with her symptoms?
B. Chronic kidney disease
C. Aortic dissection
D. Takayasu arteritis
E. Compartment syndrome

10 A 68 year-old woman presents to her family physician complaining of feelings of leg A. Air plethysmography D. This case is properly investigated using duplex ultrasonography as the first step. A diagnostic tool used to https://www.uptodate.com/contents/overview-of-lower-
restlessness and heaviness for the past 3 months. She also complains of cramping of evaluate the extent of reflux, and identify superficial, perforator or deep venous reflux extremity-chronic-venous-disease#H8
her calves, which is worse at night. She has a past medical history of hypertension,
deep venous thrombosis, type II diabetes and obesity. On physical exam, bilateral
varicose veins are observed on her calves, edematous lower legs with brown
hyperpigmentation around her ankles. What is the next best diagnostic technique?
B. Venous CT
C. Ambulatory venous pressure
D. Duplex ultrasonography
E. Magnetic resonance venography

11 A 48 year-old male complains of left lower leg swelling, cramping and heaviness which A. Radiofrequency perforator ablation C. Treatment options for reflux perforators include perforator ablation with ultrasound-guided sclerotherapy or https://www.uptodate.com/contents/overview-of-lower-
worsens with standing and relieved by walking and leg raise for 6 months. Physical endovenous methods. Not stenting. extremity-chronic-venous-disease#H5567098
examination shows 2+ edema of the left lower extremity and multiple dark blue,
dilated, tortuous veins around the left ankle and a 8x8 cm painless ulcer over the left
medial malleolus. History is significant for recurrent varicose veins, hypertension and
deep venous thrombosis. Right lower extremity is normal. Pedal pulses are palpable
bilaterally. Conservative measures have failed. Doppler ultrasonography reveals
perforating vein with reflux. Which of the following is NOT a recommended treatment
for this patient?
B. Ultrasound-guided foam sclerotherapy
C. Venous Stenting
D. Endovenous laser ablation
E. Percutaneous thermal ablation

12 A 50 year-old woman complains of on and off pain and heaviness in her right leg for A. Leg surgery or trauma E. All of the listed are risk factors for the development of venous insufficiency. High estrogen exposure and https://www.uptodate.com/contents/overview-of-lower-
the past year, which worsens with long periods of standing. Recently, she noticed pregnancy are risk factors, not nulliparity. extremity-chronic-venous-disease#H3
swelling of her right lower leg. Examination of her right lower extremity shows
telangiectasias, and dilated, tortuous veins, consistent with varicose veins. There is
also mild skin discoloration. A diagnosis of chronic venous insufficiency was made.
Which of the following risk factors do NOT contribute to the development of lower
extremity venous disease?
B. Positive family history of venous insufficiency
C. Advancing age
D. Use of oral contraceptive pills
E. Nulliparity

13 An 80 year old man comes to the office due to intermittent right eye vision loss. The A. CT scan of the head B Amaurosis Fugax is painless, rapid, transient (less than 10 minutes) of monocular vision loss. The description https://www.ncbi.nlm.nih.gov/books/NBK470528/
patient reports three episodes in the past 2 months in which he suddenly experienced of curtain falling over visual field is highly suggestive of the diagnosis. The most common etiology is retinal
“a curtain falling over the right eye” for several minutes before it spontaneously ischemia due to atherosclerotic emboli originating from ipsilateral carotid artery; therefore patients with vascular
resolved. He has no eye pain, discomfort, focal weakness, numbness or headache. risk factor like hypertension, hyperlipidemia, smoking history should be evaluated by Duplex ultrasound of the
He has a medical history of hypertension, hyperlipidemia and osteoarthritis. He was a neck. Fundoscopic examination is often normal but may show embolic plaques and retinal whitening (due to
previous smoker. b.p 140/85mmHg; pulse 74/min. Visual acuity, pupillary reflex and ischemia). Patients with amaurosis fugax and concomitant carotid artery disease have an increase risk of
fundoycopic examinations are unremarkable. Further neurologic examination shows stroke. Other causes of transients monocular vision loss are uncommon and include cardioembolic disease,
normal reflexes, motor strength and sensation. What is the next step to diagnose this giant cell arteritis, retinal vein occlusion, retinal vasospasm, papilledema.
patient?
B. Duplex ultrasound of the neck
C. Echocardiography
D. ESR
E. Ocular Tonometry

14 A 65 year old man comes for and evaluation. The patient has a history of hypertension A. Anti-platelet and statin therapy A The patient has asymptomatic bilateral carotid artery stenosis in the setting of multiple atherosclerotic risk https://www.radiologyinfo.org/en/info.cfm?pg=carotidstenosi
treated with Losartan. He is a former smoker and drinks alcohol occasionally. b.p factors like hypertension and prior smoking. All patients with carotid artery stenosis should receive intensive s
130/80mmHg and pulse is 80/min. Examination reveals a bruit in the right side of the medical therapy including antiplatelet agents like aspirin and a statin plus careful blood pressure control. Risk
neck, further examination is normal. Duplex ultrasonography of the neck arteries factors including diabetes, obesity and smoking should be managed aggressively. Further treatment is
reveals a 60 percent stenosis at the right common carotid artery bifurcation. The left dependent on patient symptomatology and severity of stenosis.
common carotid artery has a 40 percent stenosis. Which of the following is the best
next step in management of this patient’s artery abnormalities?
B. Right carotid artery stenting
C. Right carotid endarterectomy
D. Close monitoring without intervention
E. Bilateral carotid artery stenting

15 A 70 year old man comes to the office for follow-up after recent hospitalisation. 1 week A. Additional Clopidogrel therapy E The patient with temporary right sided motor weakness without evidence of infarction on MRI presents with a https://medlineplus.gov/transientischemicattack.html
ago he was evaluated in the ER for sudden onset right sided weakness that resolved TIA; workup has demonstrated high-grade carotid artery stenosis. Initial interventions for all patients with
by itself after 30 minutes. An MRI of the head was normal and the patient was carotid artery stenosis should include intensive medical management(aspirin, statin, blood pressure control)
admitted to the hospital for 24 hour observation. He has had no symptoms recurrence and counselling on lifestyle changes. Symptomatic patients with high-grade carotid stenosis(70-99 percent)
since then and feels well. Medical history includes hypertension and anxiety. He has a should be considered for carotid endarterectomy to reduce future risk of stroke. However, in patients with
20-pack-year history of smoking and quit 10 years ago. b.p 128/70mmHg and pulse is persistently disabling neurologic deficits, 100 percent occlusion of the carotid artery or life expectancy less than
74/min. Neurological exam is normal. ECG showed sinus rhythm. Doppler 5 tear are unlikely to benefit.
ultrasonography revealed a 75 percent stenosis of proximal left internal carotid artery.
Which of the following intervention is the most appropriate for this patient?

B. Ambulatory EKG monitoring


C. Comprehensive psychiatric evaluation
D. No additional intervention needed
E. Referral for left carotid endarterectomy

16 A 74-year-old man presents to the emergency department for severe right lower A. X-ray E The patient is currently showing signs of acute limb ischemia secondary to his risk factors (Type 2 Diabetes, https://www.ncbi.nlm.nih.gov/pmc/articles/PMC6723825/
extremity pain. He has a medical history of hypertension, hyperlipidemia and type 2 Hypertension, Hyperlipidemia) with symptoms specific for the disease. The patient has signs of the 6 Ps
diabetes mellitus. Physical examination is remarkable for a pale right lower extremity, (Pulseless, Pallor, Poikolothemia, Pain, Paresthesia, Paralysis). An AngioCT (fast, widely available) should be
significant weakness and sensory impairment, and absent capillary refill. Arterial the first step the physician takes if surgical intervention is considered to precisely allocate the occluded site.
Dopplers do not reveal an audible pulse. What is the next best step in diagnosis if However, anticoagulation should be given immediately which can precede diagnostic procedures to preserve
vascular intervention is considered? as much circulation as possible.
B. Angiogram
C. Doppler USG
D. Magnetic resonance Imaging
E. AngioCT

17 A 82 year old man comes to the emergency department with worsening right foot pain. A. Cutaneous vasoreactivity E The patient shows signs of critical lower limb ischemia which can proceed to acute limb ischemia if arterial https://www.ahajournals.org/doi/10.1161/CIRCULATIONAH
The pain involves the entire foot and is severe especially in the forefoot. The pain thrombosis occurs within the arteries affcted by the plaque. Atherosclerotic plaque is a common cause of A.119.039773
began a day before when the patient was lying in bed. He immediately sat up and tried chronic lower limb ischemia and increases the risk of arterial thrombosis, which is often a cause of acute lower
to alleviate the pain by dangling his foot at the bed side which helped. The pain limb ischemia.
progressively got worst and heat packs and OTC pain medications provided no relief.
He smokes 2 packs of cigarettes daily. On examination both of his legs are shiny and
hairless and the right leg is cooler to touch compared to the left. The capillary refill
time is 5-6 seconds. Which of the following caused the patient's symptoms?
B. Increased compartment pressure
C. Poplitheal vein thrombosis
D. Tobacco induced vasculitis
E. Atherosclerotic plaque disruption
18 A 78 year old female presents to the emergency department complaining of pain and A. Old age D From the listed choices, atrial fibrillation is an important cause of cardiac emboli which have travelled to the https://www.ncbi.nlm.nih.gov/pmc/articles/PMC3383153/
paresthesia in her right foot which started 6 days ago. The patient recently underwent lower limb leading to the described symptoms. Patients after cardiac procedures should be observed carefully
surgery for atrial fibrilation which went well without any complications. On physical for possible distal embolization.
examination, blood pressure was 130/90 mmHg , pulse 76 and a bmi of 25.4 kg/m2.
Weakness of pulse in the popliteal artery along with a colder sensation compared to
the left side. The patient has a history of heavy drinking in her 30s, hyperthyroidism
and osteoporosis. After examining the patient's file, which of the following risk factors
probably lead to this episode of pain?
B. Hyperthyroidism
C. History of alcoholism
D. Atrial fibrillation
E. Osteoporosis

19 A 24 year old male comes to the ER with 1 day history of right arm pain, swelling and A. Deep vein thrombosis A Upper extremity deep vein thrombosis most commonly occurs in young athletes. Symptoms are arm swelling, https://www.ncbi.nlm.nih.gov/pmc/articles/PMC5415909/#s
heaviness. He felt this pain after playing softball. He is also a weight lifter. No previous heaviness and pain. Treatment is usually thrombolysis and or 3 months of anticoagulation. Surgery may be 3title
medical history is given and he looks like a healthy young man. He does not use needed to decompress.
tobacco, alcohol or drugs and his vital signs are normal. Capillary refill is normal. What
is the likely cause of this patients symptoms?
B. Medial epicondylitis
C. Superficial thrombophlebitis
D. Lymphedema
E. Hypertrophic cardiomyopathy

20 A 20 year old heavily distressed weight lifter comes to your office with left arm A. CT C Ultrasonography with a preferred doppler reading is the first step to evaluate this patient. Noninvasive and https://www.uptodate.com/contents/primary-spontaneous-
swelling, weakness and edema. He has been lifting weights and taking boxing classes cheap to check for the pathology. upper-extremity-deep-vein-
for an upcoming boxing event he is participating in. Vitals are normal. Physical exam thrombosis?search=paget%20schroetter&source=search_r
is normal except for edematous arm and visible dilated veins. What is the first step in esult&selectedTitle=1~11&usage_type=default&display_ra
management for this patient? nk=1#H239064139
B. Xray of upper limb
C. Ultrasonography
D. Emergent surgical fixation
E. Compress stockings and reassurance

21 A 28 year old man comes in to the doctors office with pain/swelling of the left arm after A. Thoracic outlet syndrome compressing artery E Paget-Schroetter - is a form of upper extremity deep vein thrombosis - can be confused with thoracic outlet https://www.ncbi.nlm.nih.gov/pmc/articles/PMC5778512/
extensively working in his garden. He says that this is the first time something like this syndrome but TOS could also be compression of artery and the brachial plexus. Paget-schroetter comes from
has ever happened to him. Vital and physical examination is otherwise unremarkable. repetitive movement of the upper limb which is heavily done for example in prolonged gardening activity.
What is the most likely diagnosis?
B. Lymphedema
C. Superior vena cava syndrome
D. Pancoast tumor
E. Paget-Schroetter

22 4 days after undergoing a craniotomy and evacuation of a subdural hematoma, a 55 A. Iliac vein stenting C An IVC filter is indicated in this patient to prevent a major PE. It is indicated in patients with high risk or already https://www.webmd.com/dvt/ivc-filter-for-dvt#1
year old man has severe pain and swelling of his right leg. He has chills and nausea. having thromboembolism (confirmed pulmonary embolization events) who have absolute contraindications to
He has type 2 diabetes mellitus and chronic kidney disease, and was started on anticoagulation (recent surgery, intracranial hemorrhage, active bleeding), or complications from
hemodialysis 2 years ago. Prior to admission, his medications were insulin, enalapril, anticoagulation, as well as those who have not responded to anticoagulation therapy.
atorvastatin. His temperature is 38.5C, pulse is 110/min and BP is 130/80 mm hg.
Examination shows a swollen, warm and painful right calf. The peripheral pulses are
palpated bilaterally. Cardiopulmonary examination shows a recurrence of pulmonary
embolic events. Lab studies: Hbg: 10 g/dl Leukocyte count: 12,000/mm3 Platelet
count: 230,000/mm3 Glucose: 88 mg/dl Creatinine: 2 mg/dl Which of the following is
the most appropriate next step in treatment?
B. Unfractionated heparin
C. IVC filter
D. LMWH
E. Surgical thrombectomy

23 3 days after delivery, a 36 year old gravida 1, para 1 woman presents with fever and A. Unfractionated heparin A The patient presents with fever, leg pain, warm, red and edematous, as well as after delivery, which suggests a https://www.ncbi.nlm.nih.gov/pmc/articles/PMC1305303/
pain in her left leg. Pregnancy was complicated by premature rupture of membranes. postpartum DVT. US confirms it. Pregnancy is a hypercoagulable state that is accompanied by endothelial
The child was delivered at 35 weeks of gestation by lower transverse C-section. The injury during surgery and stasis due to immobilization after delivery. Smoking also increases the risk of DVT, as
patient has smoked a pack of cigarettes daily for 5 years and continued to smoke do delivery by C-section and age > 35 years. Unfractionated heparin, followed by a constant infusion for 4-5
during pregnancy. Her temp is 39C, pulse 112/min and BP is 112/83 mm hg. days is the initial treatment of choice because of immediate effect and safety during breastfeeding.
Examination shows an edematous, red and warm left leg. The uterus is nontender and
palpated at the umbilicus. US of the left leg shows an incompressible left popliteal
vein. Which of the following is the most appropriate initial step in management?
B. Fasciotomy
C. Embolectomy
D. Penicillin
E. Warfarin
24 A 30 year old woman comes to the emergency department with chest pain, SOB and A. Thrombus is the left atrium on TEE B The patient has symptoms of PE with pulmonary infarction, which occurs when there is embolism of the small https://www.ncbi.nlm.nih.gov/pmc/articles/PMC6039809/
cough with bloody sputum for the past 3 hours. The patient has a history of segmental arteries. It presents with pleuritic chest pain and hemoptysis due to an inflammatory response in the
hypertension. She has smoked 2 packs of cigarettes daily for the past 12 years. She lung parenchyma. The typical appearance of an infarct on CT is a wedge shaped defect that doesn’t fill with
drinks one glass of wine per day. She is taking enalapril and an oral contraceptive. her contrast.
temp is 37.9C, pulse 105/min, RR 22/min and BP is 103/70 mm hg. O2 saturation is
91% on room air. Physical examination shows decreased breath sounds over the left
lung base. Extremities are warm with normal pulses. Further evaluation of this patient
is most likely to show which of the following findings?
B. Wedge shaped filling defect on chest CT
C. Diffuse ST elevations on ECG
D. Bilateral infiltrates on chest x-ray
E. Tracheal deviation on chest x-ray

25 A 64 year-old man underwent a femoral percutaneous coronary intervention to place a A. Abscess B This patient presents with pain in the inguinal area probably due to the percutaneous intervention. The https://www.mayoclinic.org/diseases-
drug-eluting stent in the left circumflex artery, after having experienced retrosternal continuous murmur and palpable thrill suggest arteriovenous fistula. A pseudoaneurysm would be pulsatile, conditions/arteriovenous-malformation/diagnosis-
chest pain intermittently for several weeks. A week after the procedure, the patient nerve damage is excluded because of lack of numbness and weakness, DVT isn’t a complication of a treatment/drc-20454895
comes back complaining of continuous discomfort in the groin area for a week. On percutaneous intervention, and the patient has no fever which rules out abscess.
examination, there is slight swelling in the groin with a palpable thrill and continuous
murmur on auscultation. Vital signs, heart sounds and lung auscultation are normal.
The patient denies numbness or weakness in the leg. Which of the following is most
likely the cause of this patient’s symptoms?
B. Arteriovenous fistula
C. Deep vein thrombosis
D. Iatrogenic nerve damage
E. Pseudoaneurysm formation

26 A 63 year-old woman presents to the ER one week after undergoing a femoral A. Incision and drainage B The pulsatile mass under the skin a week after a percutaneous intervention suggests a pseudoaneurysm https://www.ncbi.nlm.nih.gov/books/NBK542244/
percutaneous coronary intervention with pain in the inguinal area. The skin over the formation. The best next step in management is to do either a duplex USG or a CT angiogram to make the
inguinal area is slightly erythematous, with a pulsatile mass right under. Her vital definitive diagnosis and rule out for instance abscess or AV malformation.
signs, heart sounds and lung auscultation are normal. Which of the following
procedures is the best next step in management of this patient?
B. Perform Duplex USG
C. Reassurance and follow-up
D. Start antibiotics
E. Surgical management of the AV fistula

27 Patient presenting with a pseudoaneurysm in the right groin undergoes a duplex USG, A. Manual compression D USG-guided thrombin injection has a success rate of 97% to 100%. Watchful waiting is relevant only with small https://www.ncbi.nlm.nih.gov/books/NBK542244/
which confirms the diagnosis. While discussing the different treatment options, the pseudoaneurysms. USG-guided compression is not effective in obese individuals, patients on anticoagulants,
patient asks which treatment option has the highest success rate. Which of the and in pseudoaneurysms with large neck.
following has the highest percentage of success in treatment of pseudoaneurysm?
B. High dose aspirin therapy
C. USG-guided compression
D. USG-guided thrombin injection
E. Watchful waiting

28 A 67-year-old man comes to the office for a new patient evaluation after recently A. Antiplatelet and statin therapy A This patient has asymptomatic bilateral carotid artery stenosis in the setting of multiple atherosclerotic risk https://www.mayoclinic.org/diseases-conditions/carotid-
relocating to a retirement community. The patient has a history of hypertension. He is factors (ex. hypertension, prior smoking). All patients with carotid artery stenosis should receive intensive artery-disease/diagnosis-treatment/drc-20360527
a former smoker with a 40-pack-year history and drinks alcohol occasionally. BP is medical therapy including an antiplatelet agent(aspirin), a statin, and careful blood pressure control.
130/80 mmHg and pulse is 80/min. Examination reveals a bruit in the right side of the Carotid intervention is recommended for symptomatic patients with high-grade carotid stenosis (generally 70%-
neck, bu the remainder of the examination is normal. Duplex ultrasonography of the 99% for symptomatic lesions). Carotid endarterectomy is generally preferred over stenting because of lower
neck arteries reveals a 80% stenosis at the right common carotid artery bifurcation. risk of stroke and periprocedual mortality.
The left common carotid artery has a 50% lesion. Which of the following is the best
next step in management of this patients arterial abnormalities?
B. Bilateral carotid artery stenting
C. Bilateral carotid endarterectomy
D. Close monitoring with no intervention
E. Right carotid endarterectomy

29 An 80 year old male presents to the ED with a pulsating mass in his abdomen and A. X-ray E The patient is stable, the cause of symptoms is unknown. AAA is suspected - abdominal ultrasound is the best https://www.clinicalkey.com/#!/content/67-s2.0-7c5e261c-
sudden onset severe pain in his back. He has a 50 pack-year smoking history. He first choice - low risk/high sensitivity. ea6f-4cab-aae7-cb8155587799
reports that he had been diagnosed with some ‘problem with his vessels, but does not
remember exactly what, nor does he remember what medications he takes. What is
the best diagnostic step?
B. MRI
C. CT without contrast
D. CT angiography
E. Ultrasound

30 Abdominal Aortic Aneurisms are associated with many risk factors, some of which are A. Obesity D Smoking increases the rate of aneurysm enlargement by 35%. Current smokers are more than seven times https://www.clinicalkey.com/#!/content/67-s2.0-7c5e261c-
modifiable. Which is the single most important modifiable risk factor in the more likely to have an aneurysm than nonsmokers, with duration of smoking the most important variable. Each ea6f-4cab-aae7-cb8155587799
development and progression of an AAA? year of smoking increases the relative risk for development of an aneurysm by 4%.
B. Hypertention
C. High cholesterol levels
D. Smoking
E. Sedentary lifestyle
31 A 58-year-old woman comes to the physician because of a 6-month history of dull, A. Duplex Ultrasonography A Duplex ultrasonography is the first-line investigation in patients with features suggestive of chronic venous https://www.ncbi.nlm.nih.gov/pmc/articles/PMC5574075/
persistent pain and swelling of her right leg. The pain is worse at the end of the day insufficiency (CVI), as seen here. Evidence of venous reflux on duplex ultrasonography confirms the diagnosis
and is relieved by walking or elevating her feet. Two years ago, she developed acute of CVI. Duplex ultrasonography can help determine the course of treatment by identifying which valves are
deep vein thrombosis in her right calf after a long flight, which was treated with incompetent and assessing the patency of the previously thrombosed deep veins.
anticoagulants for 6 months. Physical examination shows 2+ pitting edema of her right
leg. The skin around the right ankle shows a reddish-brown discoloration and multiple
telangiectasias. She has dilated varicose veins in the right leg. Which of the following
is most likely to establish the diagnosis?
B. ABI
C. D-dimer assay
D. CT + contrast
E. Phlebography

32 A 60-year-old man comes to the emergency department because of sudden, A. Venous thromboembolism D Acute limb ischemia is most commonly due to thrombosis of a previously patent artery but can also result from https://www.ncbi.nlm.nih.gov/pmc/articles/PMC6326052/
worsening pain in his right calf and foot that started 30 minutes ago. He also has a an embolus, dissection, trauma, and stent or graft occlusion. Sudden onset of pain, paresthesia, and weakness
tingling sensation and weakness in his right leg. He has had no similar episodes, in a patient with no prior history of claudication or trauma makes arterial embolization the most likely underlying
recent trauma, or claudication. He has type 2 diabetes mellitus and was diagnosed cause. Most emboli originate from thrombus formation in the heart, a complication of incomplete
with hypertension 20 years ago. His sister has systemic sclerosis. He works as an atrial/ventricular emptying in cardiac arrhythmias (e.g., atrial fibrillation), valvular heart disease, or ventricular
office administrator and sits at his desk most of the day. He has smoked one and a aneurysms (e.g., following recent anterior MI).
half packs of cigarettes daily for 30 years. Current medications include metformin and
lisinopril. His pulse is 110/min and irregular, respirations are 16/min, and blood
pressure is 140/90 mm Hg. His right leg is pale and cool to touch. Muscle strength in
his right leg is mildly reduced. Pedal pulses are absent on the right. Which of the
following is the most likely underlying cause of this patient's symptoms?
B. Popliteal artery aneurysm
C. Arterial Vasospasm
D. Arterial embolism
E. Atheroembolism

33 A 65-year-old woman comes to the physician with a 3-month history of progressively A. DVT D Presence of PAD is a very strong predictor of co-existing coronary artery disease, as both diseases have a https://www.ncbi.nlm.nih.gov/books/NBK430745/
worsening right calf pain. She reports that the pain occurs after walking for about 10 similar pathogenesis (atherosclerosis) and shared risk factors (e.g., smoking, hypertension, hyperlipidemia,
minutes and resolves when she rests. She has hypertension and hyperlipidemia. She diabetes). Compared to healthy individuals, patients with PAD are at significantly increased risk for myocardial
takes lisinopril and simvastatin daily. She has smoked two packs of cigarettes daily for infarction and stroke. Evaluation and management of underlying coronary artery disease is recommended for
34 years. Her pulse is 78/min and blood pressure is 142/96 mm Hg. Femoral and patients with PAD, and risk factors should be addressed.
popliteal pulses are 2+ bilaterally. Left pedal pulses are 1+; right pedal pulses are
absent. Remainder of the examination shows no abnormalities. Ankle-brachial index
(ABI) is 0.65 in the right leg and 0.9 in the left leg. This patient is at greatest risk of
which of the following conditions?
B. Acute mesenteric ischemia
C. Limb amputation
D. Acute MI
E. AAA

34 A 74-year old man comes to the emergency department for severe, acute right leg A. Heparin and embolectomy A Patient has acute limb ischemia and has a threatened extremity due to severe pain, decreased motor function. https://www.ncbi.nlm.nih.gov/pmc/articles/PMC6326052/
pain. Symptoms started suddenly four hours ago. He is treated for hypertension. He These symptoms require emergent revascularization. Patient likely has afib and has an intra atrial clot
has a 30 pack year smoking history. Pulse is 100/min and irregulary ireregular. Right formation that emobolized into right common femoral artery. Herparin to reduce further clot formation, balloon
leg is cool to the touch, with decreased femoral, popliteal, posterior tibial, and dorsalis catheter to relieve occlusion and restore blood flow.
pedis pulses. Moderate weakness in the right leg is observed. Whats your
management?
B. Thrombolysis
C. Amputation of affected leg
D.Surgical bypass of affected vessel
E. Percutaneous transluminal angioplasty

35 Previously healthy six month old child is brought to emergency department because A. Raynaud E Previous healthy child presents with hair tourniquet syndrome, which usually affects infants when a hair or https://www.ncbi.nlm.nih.gov/pmc/articles/PMC2941260/
her parents say she has not stopped crying for five hours. She has not eaten during thread becomes spontaneously tied around a toe and tightenes when the child moves. Worsening edema will
this time and parents are unable to calm her down. No trauma is reported by the cause arterial obstruction and ischemia distal if the hair/thread is not removed.
parents. Examination shows a reddend and swollen second toe on the left foot. On
closer examination, there seems to be a distinct ring like constricture of that toe. What
is likely diagnosis?
B. Ingrown toe nail
C. Sickle cell dactylitis
D. Insect bite
E. Hair tourniquet syndrome

36 An 8 year old patient with a history of nephrotic syndrome presented to the emergency A. Amputation of the affected limb C Treatment of acute limb ischemia should start with Heparin unless contraindications are present. Venous https://www.ncbi.nlm.nih.gov/pubmed/29655809
department with right foot pain. The patient's mother described intermittent pain that thromboembolic complications are frequently caused by nephrotic syndrome, while arterial thrombosis has
woke her son up from sleep and was accompanined by the foot turning purple and rarely been reported.
becoming cold to the touch. Physical exam showed capillary refill time of 10 seconds
on right foot, and less than 2 seconds on the left foot. Ankle-brachial indicies were 0.0
on the right, and 0.96 on the left. What is your next step in management?
B. Warfarin
C. IV Heparin
D. High-dose aspirin therapy
E. Thrombolysis

37 25 year old man comes to the office due to 3 days of right arm pain, heaviness and A. DVT A Spontaneous upper extremity DVT occurs in young athletic men who lift weight or grange in activities with https://www.ncbi.nlm.nih.gov/pmc/articles/PMC5415909/
swelling, symptoms are worse with activity and elevation above he head and improve repetitive overhead arm motions.
with rest. Patient doesn’t have any chronic diseases, but several hours prior to the
onset of symptoms he pitched 9 innings in a baseball game. There is swelling and
erythema of the left arm from the elbow to the shoulder. What is the diagnosis?
B. hypertrophic cardiomyopathy
C. lymphedema
D. medial epichondylitis
E. lateral epicondylitis

38 64 year old male with a 20 year smoking history who was diagnosed with lung cancer, A. Chest MRI B Peripherally inserted central catheters increase the risk of upper extremity DVT, especially in those with https://www.ncbi.nlm.nih.gov/pmc/articles/PMC5415909/
comes to the office due to pain and redness and swelling in his right arm. His temp is malignancy or other hyper coagulable states.
36 C, BP 130/80 mm Hg, respirations 13/min. Due to the course of chemotherapy he
had a central venous catheter placement. What is the next step?
B. USG with doppler of upper extremities
C. ECG
D. Warm compressions and NSAIDs
E. Venography

39 70 year old man comes to the emergency department with 2 days of bright blood per A. LMWH A Active bleeding - most likely due to warfarin overdose. Switching to a safer and easier to control anticoagulant https://en.wikipedia.org/wiki/Inferior_vena_cava_filter
rectum. He feels dizzy and fatigued. Patient has moderate COPD and requires is the first choice of management.
bronchodilators. During the previous hospitalization he was found to have a left
femoral and popliteal vein thrombosis, was discharged on warfarin. His BP is 80/60
mm Hg and pulse is regular and 115min. Mucous membranes are pale. There is
swelling of right ankle. Warfarin is stopped and FFP is given. What is the best
management for this patients with DVT?
B. Statin
C. TPA
D. Clopidogrel
E. Aspirin

40 A 65 years old male complains of sudden onset of severe pain in the lower part of his A. Venous thrombosis E Acute limb ischemia is most commonly due to an embolism or arterial thrombosis. Symptoms of sudden onset https://en.wikipedia.org/wiki/Acute_limb_ischaemia
right leg. The pain began a few hours before admission. The patient took NSAIDS to pain, weakness, paralysis, pulseless lower limb with no prior history of claudication or any sort of trauma makes
relieve the pain but stated that the pain persisted. The pain is still persistent despite arterial embolization the best answer. Most often emboli originate from a thrombus that formed in the heart.
lack of movement. The patients' left limb is cold, pale , painful, pulseless and he is This can be a complication of cardiac arrhythmias, valvular heart disease as seen in this patient or ventricular
unable to move it. The patient has a 30 year history of smoking and has diabetes. The aneurysms.
patient has also had a valvular replacement. What is the most likely underlying cause
of this patient’s symptoms?
B. Arterial Vasospasm
C. Atherosclerotic narrowing of the arteries
D. Inflammation induced thrombosis
E. Arterial embolism

41 A 61 year-old man comes to the ER because of severe right leg pain that occurred 2 A. Echocardiography A Echocardiography is useful for diagnosing cardiac causes of arterial embolisms. They most of arise due to a https://www.ncbi.nlm.nih.gov/pmc/articles/PMC6723825/?fb
hours ago and is consistent. The patient's symptoms started suddenly while he was atrial fibrillation. Acute limb ischemia is most often caused by arterial thrombosis, which can be secondary to clid=IwAR27GeKLI66Ttde1mSopA9Op9uU6JoaheXpUAr0
doing household chores. He has no history of leg pain at rest or with exertion. He has super of atherosclerotic plaque or thrombosis. This patient presented with an irregular pulse no history of TI7tPT9QbYRmoh4xInz4
hypertension. Temperature is 37 C, pulse is 105/min and irregular, and blood pressure claudication making A-fib the most likely reason for the ALI.
is 140/90 mm Hg. The left lower extremity is cool and tender to touch. The popliteal
and pedal pulses are decreased on the right. Sensation is diminished.
Ultrasonography confirms an embolism in the right common femoral artery. What
additional diagnostic technique is appropriate in this case?
B. ESR
C. Biopsy of the superficial vein
D. Ankle brachial index
E. Manometry

42 A 75 year-old woman comes to the ER because of sudden, severe pain in her left A. Receptors of the sympathetic nervous system D Inhibition of the synthesis of vitamin K-dependent factors by warfarin is the best treatment in patients with https://www.ncbi.nlm.nih.gov/pmc/articles/PMC6723825/?fb
lower leg. She has also had worsening tingling. She claims to have never had such suspected A-fib to prevent thromboembolic events such as acute limb ischemia. clid=IwAR27GeKLI66Ttde1mSopA9Op9uU6JoaheXpUAr0
pain in her leg. Over the last couple months, she has occasionally had episodes of TI7tPT9QbYRmoh4xInz4
palpitations. She has hypertension and type 2 diabetes mellitus. Her pulse is 90 per
min and her blood pressure is 130/80 mm Hg. Physical examination shows a cool and
pale left leg with delayed capillary filling. Muscle strength and tone in the left calf and
foot are reduced. Pedal pulses are absent on the left. Inhibition of which of the
following would have most likely prevented this patient’s condition?
B. Receptors for platelet aggregation
C. Voltage gate sodium channels
D. Synthesis of vitamin K dependent factors
E. Voltage gated potassium channels
43 A 76-year old man comes to the doctor because of a 6 month history of abdominal A. Accumulation of foam cells in the tunica intima E Degenerative aortic aneurysms are the most common type of aneurysms. They occur when the connective https://www.nhlbi.nih.gov/health-topics/atherosclerosis
pain that radiates to the back. He has smoked one pack of cigarettes daily for 40 tissue and muscular layer of the aorta weakens and gets thinner, which increases the chances that it will break
years. His blood pressure is 148/82 mm Hg. Abdominal examination shows (called rupture) or that the layers of tissue will tear (called dissection). Often, degenerative aortic aneurysms
generalized tenderness and a pulsatile mass in the periumbilical region on deep happen in the abdomen or the descending thoracic aorta. MMP-mediated loss of elastin, a hallmark of AAA, is
palpation. Further evaluation of the affected blood vessel is most likely to show which believed to lead to progressive dilatation of the aorta.
of the following as the major contributing factor?
B. Obliterative inflammation of the vasa vasorum
C. Necrotizing inflammation of the vessel wall
D. Formation of giant cells in the tunica media
E. Fragmentation of elastic tissue in the tunica media.

44 A 52 year old woman comes to the emergency department because of a 2 hour history A. Venous thrombosis C Patient presents with acute limb ischemia, which is commonly caused by arterial embolism. This patients https://www.uptodate.com/contents/clinical-features-and-
of severe pain, numbness and weakness in her right lower leg. One week ago she had syncope, heart racing and tachycardia with irregular pulse suggest arterial fibrillation. Blood stasis in atria or in diagnosis-of-acute-lower-extremity-ischemia
an episode of syncope that was preceded by severe heart racing. She has no history the ventricles results in the formation of thrombi which can embolize into systemic circulation and occlude
of illness and takes no medications. Pulse is 110 / min and irregular. Blood pressure is peripheral arteries.
124/82. The right leg below the knee is pale and cold to touch. Popliteal and pedal
pulses are absent on the right side. Which of the following is the most likely cause of
these symptoms?
B. Popliteal atery aneurysm
C. Arterial embolism
D. Atherosclerotic narrowing of the artery
E. Atheroembolism

45 Which ONE of these statements about abdominal aortic aneurysms (AAA) is correct? A. They mostly present in elderly women B. Abdominal aortic aneurysms (AAA) 90-98% are infrarenal. The aneurysm originates distal to the renal arteries https://www.ejves.com/article/S1078-5884(18)30698-
(there is nonaneurysmal aorta distal to the origins of the renal arteries). 1/fulltext#sec11.3
B. They are mostly infrarenal in location
C. Once they exceed the size of 5cm (diameter) - must be
treated within 12 hours for a successful outcome
D. Diabetes is an important risk factor
E. Smoking does not increase the chances of rupture

46 A 72 year old male patient who is generally fit and well, attends for a health A. obtain an abdominal CT B. Men ages 65-75 and with a smoking history have been identified to be at the highest risk thus an abdominal https://www.ejves.com/article/S1078-5884(18)30698-
maintenance exam. His medical history is significant for hypertension and US is recommended as US is also low cost and feasible means of testing. 1/fulltext#sec2.2.2.2
osteoarthritis. He has a 20 pack year smoking history, however he quit 30 years ago.
He is a retired plumber. His friend recently suffered from an abdominal aortic
aneurysm and is wondering if he should be checked for this also. What is the next
most appropriate step for this patient?
B. obtain a abdominal ultrasound
C. obtain an abdominal MRI
D. obtain an echocardiogram
E. no further testing needed

47 A 63 year old lady comes into the office with leg swelling that is particular A. daily furosemide C. Pt suffers from chronic venuous insufficiency due to incompetence of venous valves. It is typically worse in https://www.uptodate.com/contents/chronic-venous-
bothersome in evenings. It has gradually worsened over the last year. She smoked a evenings due to prolonged standing in day. Initial treatment is leg elevation and compression therapy. disease-beyond-the-basics#H10
pack of cigarettes a day for 30 years and does not drink alcohol. BMI is 32kg/m2. She
has 2+ pitting edema in lower extremities and dilated tortuous superficial veins. Small
ulcer is noted on left medial ankle. All peripheral pulses are palpable. Which is most
likely to relieve this patients current symptoms?
B. dietary sodium restriction
C. compression therapy
D. smoking cessation
E. high dose aspirin therapy

48 A 78 year old patient who is a lifetime non-smoker suffered a syncopal episode whilst A. Antibiotics, fluid resuscitation and admission to intensive C. Despite the patient not having severe chest pain, his symptoms are evidence of mediastinal widening https://www.ejves.com/article/S1078-5884(19)31515-
gardening. He reports chest and neck pain, over the last week he also had chest care alongside his x ray findings and pericardial effusion are concerning for acute aortic dissection. 1/fulltext#sec3.9.6
tightness, cough and whitish sputum production. He has significant hx for Hemodynamically stable patients CT angiography is recommended.
longstanding hypertension and type 2 DM. ECG is normal. Bedside transthoracic
echocardiogram shows small pericardial effusion and x-ray reveals slight mediastinal
widening. What is the best next step in management?
B. Cardiac catheterization
C. CT angiography
D. Immediate emergency pericardiocentesis
E. Carotid US Doppler
49 An 88 year old man experiences severe right calf pain several hours after undergoing A. Venous thrombosis E. Clinical features of compartment syndrome are pain out of proportion to injury, pain increased on passive https://www.ejves.com/article/S1078-5884(19)31515-
right femoral artery embolectomy. He also has a burning sensation in posterior aspect touch, tense swelling and paraesthesia. It is often a consequence of revascularization after acute limb 1/fulltext#sec3.9.6
of his right leg. He has persistent AF and hypertension. He suffers from Diabetic ischemia.
nephropathy also. Right calf is swollen, tense and exquisitely tender with pain worse
with passive dorsiflexion of the ankle. Skin is shiny and cool to touch. dorsalis pedis
and posterior tibial pulses are palpable. which of the following is the most likely cause
of patients symptoms?
B. Necrotising fascitis
C. Recurrent embolism
D. Vaso-occlusive crisis
E. Compartment syndrome

50 There are several prognostic risk factors for peri-operative mortality following an open A. Cardiac arrest D. Alcohol consumtion has not be recorded to be associated with increased mortality peri-operatively, CHF, https://www.ejves.com/article/S1078-5884(18)30698-
rAAA repair. Which one of the following is NOT a factor? severe anemia, cardiac arrest, deteriorated consciousness, renal impairment and pre-operative severe 1/fulltext#sec4.3.4
haemodynamic instability are currently the most recognised factors
B. Deteriorated consciousness
C. Renal impairment
D. Increased levels of blood alcohol
E. Congestive heart failure

51 After aortic aneurysm surgery, an increased incidence of deep venous thrombosis A. In patients of asian decent E. A prolonged postoperative stay on the intensive care unit alone can increase the risk of a pulmonary embolism https://www.ejves.com/article/S1078-5884(18)30698-
(DVT) was observed in which ONE of the following patient subsets? - the additional factors of prolonged stay induces stasis of blood and increases the chances of DVT occurring. 1/fulltext

B. In male patients
C. In patients with ischemic necrotic lesions present on
their limbs
D. In patients with coexisting diabetes
E. In patients with prolonged postoperative stay on the
intensive care unit

52 A 66-year-old retired teacher comes to hospital with a 3-week history of severe pain A. Weight reduction C Smoking and diabetes are major risk factors for chronic limb-threatening ischemia. Minor risk factors include https://www.ejves.com/article/S1078-5884(19)30380-
in his right leg. He took ibuprofen with little improvement. The pain aggravates in the HTN, obesity, and sedentary lifestyle. 6/pdf
morning and is relived by lowering the limb off the side of the bed. He smokes 1 pack
and drinks 2 bottles of alcohol per day. Physical exam shows a temperature of 37.3 C,
BP 149/110 mmHg, PR 78/min, and RR of 18/min. His BMI is 23 kg/m2. Capillary refill
is 7 seconds. There is no hair on his right leg. Which of following is the best for disease
prevention?
B. Reduce salt intake
C. Smoking cessation
D. Stop alcohol intake
E. Thrombolysis administration

53 A 56-year-old man complains of claudication in buttocks and thighs. He complains A. Blue toe syndrome C Leriche’s syndrome or aortoiliac occlusive disease is the diagnosis. Classically, it is described in male patients as https://www.uptodate.com/contents/clinical-features-and-
that he could not have sexual intercourse because of erectile dysfunction. Physical a triad of the following signs and symptoms: diagnosis-of-lower-extremity-peripheral-artery-disease
examination is unremarkable except for decreased femoral pulses. What is the best claudication of the buttocks and thighs
explanation of his symptoms? absent or decreased femoral pulses
erectile dysfunction
B. Chronic venous insufficiency
C. Leriche’s syndrome
D. Excessive exercise
E. Iliac artery aneurysms

54 A 54-year-old woman is diagnosed with chronic limb-threatening ischemia. She has a A. Alcohol consumption E DM and CKD can cause falsely negative results of ABI due to incompressible vessel pathology. https://www.ejves.com/article/S1078-5884(19)30380-
history of diabetes mellitus and hypertension for 28 years. She smokes and drinks 6/pdf
alcohol daily, Her BMI is 34 kg/m2. Upon investigation, her ankle-brachial index is
normal while toe-brachial index is very low. What is the explanation for the difference
between the 2 indices?
B. Smoking history
C. Obesity
D. Hypertension
E. Diabetes mellitus
55 A 55-year-old man with history of stable angina comes to the hospital with an ulcer A. Thromboxane A2 inhibitors C HMG-CoA reductase inhibition is the mechanism of action for statin drugs. https://www.ejves.com/article/S1078-5884(19)30380-
on his big toe of the left leg. The ulcer became present about a week ago. He 6/pdf
complains of left leg pain at rest which began 3 weeks ago. The patient was
prescribed few medications including atorvastatin. What is the mechanism of this
drug?
B. Bile acid sequestrants
C. HMG-CoA reductase inhibitors
D. Squalene epoxidase inhibitors
E. Beta-2 blockers

56 A 76-year-old woman is admitted to the hospital with a large necrotic area on her A. Amputation A Offer primary amputation to CLTI patients who have a pre-existing dysfunctional or unsalvageable limb, a poor https://www.ejves.com/article/S1078-5884(19)30380-
dorsal and lateral right foot. The ulcer has been present for approximately 3 weeks. functional status (eg, bedridden), or a short life expectancy after shared decision-making with the patient and 6/pdf
She has a history of diabetes mellitus, hypertension, and stroke. She is bedridden health care team.
under supervision of her daughter. She claims that her right leg pain often wakes her
up at night. Which is the next step of management?
B. Thrombolysis
C. By-pass surgery
D. Angioplasty
E. Biologic therapy

57 A 64-year-old man with history of atherosclerosis comes to the hospital after 4-weeks A. Chronic limb ischemia C Critical limb ischemia is the presence of chronic lower limb ischemia in combination of rest pain, gangrene, or https://www.ejves.com/article/S1078-5884(19)30380-
of left leg pain and numbness. The pain usually occurs after waking up. When patient a lower limb ulceration > 2 weeks duration. 6/pdf
sits, his left leg increases in redness. Upon examination, small ulcer is detected on left
big toe. What is the diagnosis?
B. Acute limb ischemia
C. Critical limb ischemia
D. Diabetic neuropathy
E. Nerve root compression

58 A 57 year-old woman comes with left foot pain that occurred at rest for 2 weeks. She A. Perform computed tomography angiography B ABI is the best differential diagnostic method. ABI <0.90 abnormal, rest pain usually appears at <0.30. CT and https://www.ejves.com/article/S1078-5884(19)30380-
informs that the pain used to occur after walking for some distance. She has a history MRA should be done before revascularization is planned. 6/pdf
of hypertension and chronic kidney disease. What is the next step of diagnostic
workup?
B. Perform ABI (ankle brachial index)
C. Perform magnetic resonance angiography
D. Perform catheter directed thrombolysis
E. Perform lower extremity X-ray

59 What is the most common location for the formation of atherosclerotic plaques in the A. Dorsalis pedis artery D Chronic limb ischemia most commonly involves the superficial femoral artery - that is also the reason for most https://www.ejves.com/article/S1078-5884(19)30380-
arteries of the lower limb? commonly causing claudication of the calf muscle. 6/pdf
B. Posterior tibial artery
C. Deep femoral artery
D. Superficial femoral artery
E. Superior gluteal artery

60 A 61 year old is admitted to the ER with sudden onset of a painful, cold, white right A.Abdominal aortic aneurysm C The presentation is that of an embolic episode which occluded flow to the femoral artery. Eighty percent of https://www.ejves.com/article/S1078-5884(19)31515-
leg. His radial pulse rate is 86 bpm and its rhythm follows no discernable pattern emboli have a cardiac cause (AF, MI and ventricular aneurysm). 1/fulltext#sec3.9.6
throughout 30 seconds of palpation. Abdominal examination is normal. No pulses are
palpable in the right leg and ankle Doppler signals are absent. An ECG confirms the
arrythmia but shows no signs of acute heart ischaemia. Which is the single most likely
cause of the signs and symptoms?
B.Aorto-iliac dissection
C.Atrial fibrillation
D.DVT
E.MI

61 A 70-year-old woman develops a cold, painful, right leg. No pulses are palpable A History of 40 pack-years of smoking E Symptoms of acute limb ischemia - (all may not be present) - Pain: absent in 20% of cases, Pallor: within a few https://www.ejves.com/article/S1078-5884(19)31515-
throughout the right leg. She has several risk factors for atherosclerotic disease. A hours becomes mottled cyanosis, Paresthesia: light touch lost first then sensory modalities, Paralysis: most 1/fulltext#sec1.2.2
medical student asks how best to differentiate between acute and chronic ischaemia. important, heralds impending gangrene, Polar/Poikilothermia (cold), Pulselessness: not reliable. Note:
Which is the single most appropriate advice? paresthesia and paralysis are not present in chronic limb ischemia!
B History of intermittent claudication
C History of thrombotic stroke
D Presence of femoro-popliteal bypass scar in the left leg
E Presence of paresthesia and paralysis of the affected limb
62 A 65-year-old woman with a 40-pack-year smoking history and type 2 diabetes A Amputation D Most important modifiable risk factors: smoking, diabetes, hypertension, sedentary life-style, https://www.ejves.com/article/S1078-5884(19)30380-
presents with cramp-like pain in her right calf after walking 500 metres, relieved by hypercholesterolemia, 6/fulltext#sec5.1
rest. Her symptoms are aggravated by walking up steep hills. Which is the single most
appropriate management?
B Diagnostic angiography
C Endovascular treatment
D Modification of risk factors
E Reassure and follow up in 6 months

63 A 55-year-old man who smokes 40 cigarettes per day presents with a history of pain in A .Left common iliac artery occlusion E An occlusion is a complete blockage of an artery whereas a stenosis implies narrowing of the vessel. Calf https://www.ejves.com/article/S1078-5884(19)30380-
his left calf after walking 100 metres. The pain subsides with rest. He has no claudication is most commonly caused by superficial femoral arterial disease. Absent popliteal and pedal 6/fulltext#sec5.1
symptoms in his thigh or buttock. He has a palpable femoral pulse but no popliteal or pulses suggest that significant symptomatic arterial disease exists proximal to the level of the popliteal artery
pedal pulses are present. Neurological examination is normal. He undergoes duplex
Doppler ultrasound. Which is the single most likely finding of this investigation?

B .Left internal iliac artery stenosis


C .Left posterior tibial artery occlusion
D. Left profunda femoris artery stenosis
E. Left superficial femoral artery occlusion

64 A 69-year-old man is referred to the vascular clinic after an abdominal aortic A Abdominal aortic aneurysms are associated with tobacco B Abdominal aortic aneurysms are associated with hypertension, smoking, family history, but not with diabetes https://www.ejves.com/article/S1078-5884(10)00560-
aneurysm was detected coincidentally on ultrasound examination. The patient is smoking, hypertension, family history, and diabetes mellitus. CT would not be an appropriate screening tool due to high radiation dose and cost. Abdominal aortic 5/fulltext#sec3.2
nervous about the diagnosis, has been researching it on the Internet and has several mellitus aneurysms occur in 5% of males over 65 (they are approximately nine times commoner in men than in
questions. Which single statement is correct? women).
B Asymptomatic adominal aortic aneurysms are considered
for treatment by surgical or endovascular repair when they
reach a size of ≥5.5cm, in a patient fit for intervention

C Abdominal aortic aneurysms most commonly involve the


aorta at the level of the renal arteries
D Abdominal aortic aneurysms occur most commonly in
the female population aged over 65
E Abdominal aortic aneurysm screening is undertaken using
CT imaging

65 A 23-year-old medical student returning from her elective in Australia develops a A. Ascending venography D This student has clinical features suggestive of a deep vein thrombosis. The most sensitive, specific, and https://www.esvs.org/wp-content/uploads/2015/12/CLTI-
tender, warm, swollen right calf within 12h of her flight. She smokes five cigarettes appropriate investigation in this case is duplex ultrasound of the leg which will demonstrate occlusive Guidelines-ESVS-SVS-WFVS.pdf
daily and takes an oral contraceptive. She has no chest pain or shortness of breath. thrombus and blood flow disturbances caused by the clot. D dimers are a sensitive test for deep vein
Which single investigation is the most appropriate? thrombosis but are usually employed as part of a thrombotic screening and suspicion of a PE.
B. CT pulmonary angiography
C. D-dimer screening
D. Duplex ultrasound scan
E. VQ scan

66 A 43-year-old office worker, who has had a swollen left leg since her early teens, has A. Aorto-iliac arterial disease E This patient has a history suggestive of congenital unilateral lymphoedema (Milroy’s disease), associated with https://www.ejves.com/article/S1078-5884(18)30428-
aching in the calf at rest and has developed a 4cm shallow ulcer, with an hypoplasia of the lymphatic trunks. Long-term lymphoedema predisposes to skin changes including leg ulcers, 3/pdf
erythematous base, above her lateral malleolus. What is the most likely diagnosis? fungal infections, and increased risk of cellulitis.
B. Cellulitis
C. Deep vein thrombosis
D. Lipodermatosclerosis
E. Lymphoedema

67 A 22-year-old woman employed at the butcher’s counter at her local supermarket A. Oral prednisone B This woman is exhibiting Raynaud’s phenomenon on exposure of her hands to a cold stimulus. This occurs nine https://www.researchgate.net/publication/319678014_ES
complains that her fingers become white, then blue and cold at work. When she times more commonly in females and characteristically affects young women. The redness of her hands on VM_guidelines_-
warms her hands under the hot tap they become acutely painful developing a deep warming is caused by reactive hyperaemia, not a reperfusion injury, which is often accompanied by pain and _the_diagnosis_and_management_of_Raynaud's_phenom
red colour. Which is the single most appropriate management? paraesthesia. Initial management should be based around conservative measures including avoidance of cold enon
stimuli, stopping smoking, and wearing gloves where feasible. Second-line therapy involves treatment with
calcium channel blockers.
B. Request transfer from her current job to a different role

C. Lumbar sympathectomy
D. Oral prostacyclin
E. Nifedipine 5mg three times daily increasing to 20mg
three times daily
68 A 43 year old man came into the office complaining of headaches and occasional A. CT angiography D This man is exhibiting signs/symptoms of subclavian steal syndrome (headaches, dizziness, permanent https://www.uptodate.com/contents/subclavian-steal-
dizzines for the past 2 weeks. He had tried over the counter pain medication but it did ischemic changes to ipsilateral hand, pressure difference in arms, murmur). Initial diagnosis is made using syndrome
not relieve his headaches. A week ago he noticed skin color changes his left hand that duplex ultrasound looking for reversal of blood flow in the vertebral arteries.
have remained. On physical examination there is blood pressure difference in both
arms of 60mmHg and a murmur is heard over the left subclavian artery. What is the
first line diagnostic tool?
B. MRI
C. TEE
D. Duplex Utrasound
E. Transcranial Doppler

69 A 36 year old gym enthusiast walks into the ER complaining of severe chest pain of A Chest X-ray B This patient is presenting with a clinical picture of acute aortic dissection (cocaine use, gym enthusiast, ripping https://www.ejves.com/article/S1078-5884(18)30692-
ripping character radiating to the back. His history is unremarkable. The patient chest pain, transient elevated BP). Initial diagnosis when suspecting AAD in a hemodynamically unstable 0/fulltext#sec2.2
admits to using cocaine occassionally . Physical exam presents absent carotid pulse patient is beside TEE or TOE, versus in a hemodynamically stable patient CT angio of chest and abdomen is
and elevated blood pressure. What is the first step of management? used. CT angio is less user dependent, more readily available at some hospitals, detects true and false lumen
and potential aortic branch compromise and aids in treatment plan.
B CT angiography https://www.uptodate.com/contents/management-of-
acute-aortic-
dissection?search=aortic%20dissection&source=search_res
ult&selectedTitle=2~150&usage_type=default&display_ran
k=2
C Duplex Ultrasound
D Transoesophageal echocardiogram
E ECG

70 A 65 years old man comes to the out-patient clinic complaining of the ulcer on the left A. Brachial blood pressure measurement for both arms C A D-dimer test is not at all likely to diagnose subclavian steal syndrome. The listed tests: Brachial blood https://www.uptodate.com/contents/subclavian-steal-
index finger nail bed, intermittent color change of the left arm. He also experienced pressure measurement for both arms, CT angiography, Duplex USG, Transcranial Doppler - can be suggestive or syndrome?search=subclavian%20steal%20syndrome&sour
episodes of syncope and dizziness for past 2 years. Patient has been suffered from diagnose SSS. ce=search_result&selectedTitle=1~14&usage_type=default
uncontrolled hypertension for 12 years. Physical examination shows a neck bruit in &display_rank=1
the left subclavian region, which of the following below is least likely needed for
diagnostic workup to approach your suspected diagnosis?
B. CT angiography
C. D-dimer test
D. Duplex USG
E. Transcranial Doppler

71 A 62 years old woman comes to the outpatient clinic for the annual checkup for A. Prescribe medication for diabetes mellitus and A The majority of patients with subclavian steal syndrome is asymptomatic and the “steal” of the blood is https://www.uptodate.com/contents/subclavian-steal-
diabetes mellitus. Her height is 155 cm, weighed 76 kg. Blood test shows the level of: dyslipidemia and educate the patient on the importance of insignificant for most of the cases. The main goal for these patients is to control the current chronic diseases syndrome?search=subclavian%20steal%20syndrome&sour
Fasting glucose level 175mg/dL, Total cholesterol 230mg/dL, Triglycerides 163mg/dL, diabetes control and life style changes like DM, HTN, dyslipidemia. Surgery may only be needed if there iare significant symptoms associated with ce=search_result&selectedTitle=1~14&usage_type=default
LDL cholesterol 139mg/dL, HDL cholesterol 50mg/dL. Physical examination reveals moderate to severe stenosis of the subclavian artery. &display_rank=1
slightly diminished radial pulse of the right arm. Right brachial systolic blood pressure
is 16mmHg lower than on the left. The patient denies of any uncomfort, what is the
next step?
B. Send the patient to the vascular surgery department to
schedule angioplasty of the subclavian artery
C. Schedula an MR to rule out TOS
D. Send the patient to the vascular surgery department to
schedule open by-pass surgery
E. Schedule a CT with contrast of the aortic arch and its
branches

72 A 60 years old man was found after collapsing at work. He has been working as a A. Weight control and diet change E The symptoms of subclavian steal syndrome will be exacerbated by the increase of exercise especially of the https://www.uptodate.com/contents/subclavian-steal-
mechanical operator for 35 years and started to smoke since then. After the patient affected arm. The flow to the basilar artery will decrease due to the need to supply of the exercising arm. In syndrome?search=subclavian%20steal%20syndrome&sour
regained the consciousness, he stated that he had few similar experiences over the this case the patient should undergo smoking cessation and reduce the movement to prevent the collapse or ce=search_result&selectedTitle=1~14&usage_type=default
past 3 months especially when there was heavy workload during the day. The patient damage before the condition sorted. Weight loss and diet change, regular BP control are also needed as &display_rank=1
has the BMI of 29 kg/m2. Physical examination reveal coldness, paleness and weak hypercholesterolemia and elevated blood pressure are important risk factors of SSS.
pulse of the right arm. Glucose and lipid profiles are normal. Before sending him to
the vascular surgeon, what advise would NOT be important for this patient?

B. Smoking cessation
C. Prevent heavy exercise for the right arm
D. Regular BP control
E. Avoiding any alcohol intake
73 A 54-year-old male is sent to ER due to the third syncope in this month. The patient is A. Reverse flow in the left vertebral artery A Subclavian steal syndrome will lead to the reverse flow of the ipsilateral side of the stenotic lesion. The blood https://www.uptodate.com/contents/subclavian-steal-
obese with BMI of 34kg/m2. He has been smoking for the past 35 years. The CT is stolen from the contralateral vertebral artery which gives blood supply to the basilar artery and then to syndrome?search=subclavian%20steal%20syndrome&sour
angiography and Duplex USG shows an occlusion of left subclavian artery >85%. What posterior cerebral artery. ce=search_result&selectedTitle=1~14&usage_type=default
else can be suspected in the patient? &display_rank=1
B. Increased flow to the basilar artery
C. Decreased flow to the right vertebral artery
D. Increased flow to the posterior cerebral artery
E. Reverse flow in the right vertebral artery

74 A 35 years old woman comes to outpatient clinic complaining of claudication of her A. Increased plasma concentration of clotting factors and D In most cases Takayasu arteritis can be diagnosed with imaging showing the narrowing of the branches and https://www.uptodate.com/contents/clinical-features-and-
right arm and lightheadedness. She is under hypertensive treatment for the past 7 fibrinogen due to activation of gene transcription the suggestive symptoms (eg. Constitutional symptoms, hypertension, diminished or absent pulses, and/or diagnosis-of-takayasu-
years and oral contraceptive is taken. Medical history record reveals the patient has arterial bruits). Some of the symptoms are similar to subclavian steal syndrome (eg. Pressure and pulse arteritis?search=takayasu%20%E5%8B%95%E8%84%88%E
suffered from fatigue and muscle weakness, joint pain in multiple areas since a year difference, subclavian artery bruit). In patients with TKA, the infiltrates containing cytotoxic lymphocytes cause 7%82%8E&source=search_result&selectedTitle=1~70&usag
before. The patient lost 16kg from last year without cutting daily food intake. The injury to the arteries due to the release of the inflammatory contents. The risk of oral contraceptives causing e_type=default&display_rank=1
physical examination reveals bruits in the right subclavian and carotid artery region. thrombosis and embolism is relatively low when the patient denies usage of tobacco. Also, the
The pulse of the right arm is prominently diminished, the brachial pressure difference thromboembolism side effect of the estrogen containing oral contraceptives are much less likely happen in
between both arms is more than 25mmHg. She denies any cigarrete, drug, and arteries than in veins. The symptoms in the case are more suggestive to TKA than Neoplasm, Myasthenia
alcohol use. What is the most likely etiology causing the condition of the patient? gravis.

B. Exposure to great amount of carcinogen or familial


mutation of protooncogene
C. Antibodies blocking ACh receptors in the motor end
plate
D. Toxic T lymphocytes mediated inflammatory process
E. Congenital chromosomal anomalies

75 A 47 years old male admitted to the department due to pain and coldness of the right A. Subclavian steal syndrome B The past medical history and the current condition are suggestive to TOS. The most common etiology for TOS https://www.uptodate.com/contents/overview-of-thoracic-
hand. The right brachial systolic pressure is lower than on the left. The medical history is the chronic inflammatory process due to injury. TOS can be categorized into 3 main types: neurogenic, outlet-
reveals patient falling of off stairs 3 months ago with rib fractures. Diagnostic workup venous, and arterial. The subclavian artery is commonly affected in the arterial TOS which can be scanned by syndromes?search=thoracic%20outlet%20syndorme&sour
shows the stenosis of the right subclavian artery. Which of the flowing is the most USG or angiography. Patient’s age and information given are not enough to state subclavian steal syndrome. ce=search_result&selectedTitle=1~51&usage_type=default
probable diagnosis? Also Buerger disease affected small to medium artery. &display_rank=1
B. Thoracic outlet syndrome
C. Disseminated intravascular coagulation
D. Giant arteritis
E. Buerger disease

76 A 38 year old female is admitted to the department due to pain, dysesthesia, and A. Shoulder restraint B Neurogenic TOS is the most common form of TOS. The electrodiagnostic workup is specific but not sensitive to https://www.uptodate.com/contents/overview-of-thoracic-
numbness of the right hand. The symptoms have been present since the car accident detect nTOS. The treatment of nTOS include physical and medical therapy, if the patient fails to gain response outlet-
happened 6 weeks ago. The electrodiagnostic workup showed a positive result. from the therapies than the decompression surgery can be suggested. Anticoagulation therapy is treatment for syndromes?search=thoracic%20outlet%20syndorme&sour
Patient was put on physical therapy but did not gain a response from the therapy. venous TOS not nTOS. ce=search_result&selectedTitle=1~51&usage_type=default
Which of the following forms of management can be suggested to the patient? &display_rank=1

B. Decompression surgery
C. Anticoagulation therapy
D. Thrombolysis
E. High-dose aspirin therapy

77 A 74 years old male was sent to the hospital due to vertigo, disequilibrium, chest pain A. The reverse blood flow of vertebral artery can possibly E Coronary subclavian steal syndrome occurs when coronary artery bypass surgery is done before the symptoms https://www.uptodate.com/contents/subclavian-steal-
and left upper limb skin color changes for a day. He had a history of severe coronary be seen in the patient of stenosis of subclavian artery occur. The blood flow of internal mammary artery goes reverse to supply the syndrome?search=subclavian%20steal%20syndrome&sour
artery stenosis and coronary artery bypass surgery was done 5 years ago. Left internal ischemic axillary artery. Jaw claudication happens in giant cell arteritis. ce=search_result&selectedTitle=1~14&usage_type=default
mammary artery was utilized to bypass the blood flow. Which of the following is least &display_rank=1
likely to be correct about the patient’s condition?
B. The reverse blood flow of the left mammary artery can
possibly be seen in the patient
C. The percutaneous transluminal angioplasty of subclavian
artery before the coronary artery bypass may prevent the
issue
D. CT angiography can be done to rule out aortic dissection

E. Patient may show signs of jaw claudication


78 A 48-year-old woman comes to the outpatient clinic due to the intermittent, A. Varicosities C Chronic venous disease/insufficiency (CVD or CVI) is a problem that affects the veins of the legs. If the valves https://www.uptodate.com/contents/chronic-venous-
moderate pain of her left leg for the past 6 months. She does not recall having any within the veins fail to work properly, there is a blockage to normal flow, or the calf muscles cannot pump disease-beyond-the-basics#H1
significant injuries. She has been teaching in a primary school for 20 years. After long properly, blood can flow backwards in the veins and pool in the legs. The pooled blood can increase pressure
periods of standing, there is edema of her left leg. This swelling is reduced significantly in the veins. This can cause problems that range in severity from mild (such as a feeling of leg heaviness,
when she props her leg up while lying down or walks. By the end of the day, she feels aching, or dilated or unsightly veins) to severe (such as swelling of the leg, ankles or feet, skin color changes,
tiredness and tightness in her leg. She also reports a leg cramp at night. On physical skin rash on the leg, recurrent skin infections, and chronic ulcers). Varicose vein (a bulging of twisted vein as
examination, there is a bulging of twisted vein on the medial aspect of her left thigh describe in question) is a feature of venous disease that caused by a dilation of the veins. Pain and extremity
and a brown-colored skin near the left ankle. Her left leg is warm to touch and no swelling associated with venous disease is typically worse when standing, or when seated with the feet
eczema or ulcers. The femoral and pedal pulses are present. Duplex ultrasound dependent for prolonged periods of time, and improves with limb elevation and walking. These features
examination showed reflux (48 seconds) and the reflux started from an incompetent distinguish CVD from typical symptoms associated with lower extremity occlusive PAD. Patients with mild-to-
great saphenous vein. Which of the following is the most likely diagnosis? moderate lower extremity PAD complain of pain with walking that is relieved by rest while those with more
severe PAD may complain of more pain when the limb is elevated.
B. Peripheral artery disease
C. Chronic venous insufficiency
D. Thrombophlebitis
E. Arterial ischemia

79 A 34-year-old woman comes to a private clinic because of an unsightly aesthetic A. Increased oxidative stress leading to endothelial B Varicose veins contain an increased amount of collagen and decreased number of smooth muscle cells and https://www.ejves.com/article/S1078-5884(15)00097-
appearance of skin at the back of her left knee. She describes it as “a large visible damages elastin leading to disorganization of muscle components, disruption of elastic fibres, and fibrosis. The 0/fulltext
structure (3 mm in diameter) that can be felt and is greenish in color, protrudes from weakness of the vein wall results in dilation and enlargement of the valve ring, making the valve unable to
the surface of the skin, and it has a worm-like appearance”. As a model, this ugly work sufficiently, with reflux as the consequence.
worm is unacceptable. Apart from feeling tiredness in her legs after wearing high
heels and walking on a runway, she has no any other symptoms. Which of the
following is the most likely mechanism of this patient’s complaint?
B. Structural changes in blood vessel valve resulting in
venous reflux
C. Inflammation due to blood clot in vein
D. Blockage of the arteries supplying the lower limbs
E. Abnormal communications between arteries and veins

80 A 45-year-old woman comes to the outpatient clinic due to heaviness and swelling in A. Plethysmography E Duplex ultrasound examination (DUS) is an accurate, non-invasive technique which is based on a combination https://www.ejves.com/article/S1078-5884(15)00097-
her right leg at the end of the day. She is a scrub nurse in the operating room which of ultrasound imaging and pulsed wave Doppler with information that can be obtained on both the anatomy 0/fulltext
requires her to stand for long periods. Her mother who is a hairdresser also had and the hemodynamic features of the venous system. Anatomy, valvular incompetence, and venous
similar symptoms. On physical examination, there are greenish-purple clusters of obstruction can be easily detected using DUS. Symptomatic patients with lower extremity telangiectasias,
blood vessels that easily visible through the skin. They have a worm-like appearance reticular veins, or varicose veins should undergo further evaluation with venous duplex to identify the
and spread across the anterior calf of her right leg. Which of the following is the most presence of superficial, perforator, or deep venous reflux, which alters treatment options.
appropriate next step in management?
B. Phlebography
C. Computed Tomography Venography
D. Magnetic Resonance Venography
E. Duplex ultrasound scan

81 A 65-year-old woman presents to the outpatient clinic for evaluation of varicose vein A. Age C Older age is the most important risk factor for varicose veins and chronic venous insufficiency. Varicose vein is https://www.ejves.com/article/S1078-5884(15)00097-
on her right leg that has been slowly progressing. Her past medical history is notable more common in female adults than males. A body mass index (BMI) greater than 30 increases the risk for CVI 0/fulltext
for type 2 diabetes mellitus, hypertension, and hypercholesterolemia. Her younger significantly. Although genetic disturbances have not been found to explain the obvious heredity, many studies
brother also has varicose veins. She has no prior surgeries. She consumes alcohol have shown a correlation between a positive family history for varicose veins or venous disease and the risk of
occasionally but does not smoke cigarette. On physical examination, her blood varicose veins.
pressure is 128/86 mmHg, pulse rate is 78/min, and her BMI is 31 kg/m2. There is a
varicose vein 4 mm in diameter on the dorsal aspect of her right lower leg. Which of
the following is NOT a risk factor for patient’s varicose vein?
B. Gender
C. Diabetes
D. Family history
E. Obesity

82 A 55-year-old married woman working as a factory worker comes to the outpatient A. Skin hyperpigmentation of leg E As venous disease progresses, untreated varicose veins can lead to the development of chronic venous https://www.ejves.com/article/S1078-5884(15)00097-
clinic because of edema of her left leg for the past 3 weeks. The swelling is reduced changes such as edema after standing for long periods. Skin changes (hyperpigmentation, eczema, 0/fulltext
significantly when she elevates her leg for a couple hours and worsens progressively lipodermatosclerosis or atrophie blanche), and eventually, ulceration (this is an indication that the venous
after a long day at work. She reports heaviness and cramping in the same leg. She has disease has reached an advanced stage) may become present.
also noticed a varicose vein on her leg in the past 5 years and it has gotten larger over
time. The patient does not recall any trauma to the legs. Doppler ultrasound
examination detected the reflux of perforating vein in the left thigh. If her leg is left
untreated, it can progress to all of the following EXCEPT?
B. Lipodermatosclerosis
C. Atrophie blanche
D. Leg ulcer
E. Ischemic leg

83 A 60-year-old man was diagnosed with a varicose vein of the small saphenous vein. A. Taking prophylactic antibiotics D According to EVSV guidelines for compression after venous intervention “Post-procedural compression is https://www.ejves.com/article/S1078-5884(15)00097-
He is scheduled for an endovenous thermal ablation (EVTA). Which one of these is the recommended after superficial venous surgery, endovenous truncal ablation, and sclerotherapy” (Grade IA). A 0/fulltext
most appropriate recommendation for this man after EVTA treatment? short duration (about a week) compression after varicose vein surgery is good regarding post-operative pain,
leg volume, incidence of complications and absence from work.
B. Taking acetylsalicylic acid and oral zinc to promote
wound healing
C. Leg elevation and leg massage
D. Wearing compression stockings
E. Walking exercises

84 Which of the following is NOT a contraindication to lower extremity endovenous A. Saphenous vein incompetence A Endovenous ablation generally should not be performed in patients who have signs of acute superficial vein https://www.uptodate.com/contents/approach-to-
ablation therapy for superficial venous insufficiency? thrombosis due to the increased risk of deep venous thrombosis and/or thromboembolism. Endovenous treating-symptomatic-superficial-venous-
ablation is contraindicated in patients with acute deep vein thrombosis due to the risk of developing a new insufficiency?search=endovenous%20ablation&source=sea
thrombosis. Pregnant patients should defer vein treatments until at least six weeks after delivery due to rch_result&selectedTitle=5~150&usage_type=default&disp
prothrombotic risk and postpartum hormonal changes. Peripheral artery disease (PAD) is a relative lay_rank=5#H4119304806
contraindication to venous ablation because of the increased risk for nonhealing wounds.
B. Superficial vein thrombophlebitis
C. Pregnancy
D. Peripheral artery disease
E. Acute deep venous thrombosis

85 A 13-year-old boy was referred to the vascular surgery clinic because of varicosity in A. Deep vein thrombosis D Klippel-Trenaunay syndrome (KTS) is a rare congenital disorder characterized by capillary, venous, and https://www.ejves.com/article/S1078-5884(15)00097-
the lateral aspect of his left leg. He also complains about leg discomfort and altered lymphatic abnormalities, without significant arteriovenous shunting. Patients with this condition are 0/fulltext
gait. He said, “His leg is too heavy to move around with and he could no longer wear characterized by the clinical triad of port wine stain (“naevus flammeus”, are present from birth but may not
the old shoes”. He was delivered at home and his mother stated that he has multiple be visible until a few days after delivery. They are multiple, affecting the lower limb and can spread to the
reddish-pink macules on his chest and leg that are present since birth. On physical buttock or chest), varicose veins, and soft tissue and/or bone hypertrophy (occasionally hypotrophy). KTS must
examination, there was a difference in the length of legs and the right lower limb be considered if extensive varicose veins are encountered in a young patient, especially if unilateral and in an
showed significant enlargement of the soft tissue. An x-ray film of the legs shows a atypical distribution (lateral leg). Clinical findings are usually present at birth, but in some cases they may not
limb length discrepancy with the left lower extremity was about 2 cm longer than the be fully apparent. Complications of KTS include clotting disorder and chronic thromboembolism, bleeding,
right one. Doppler ultrasound was done, there was a dilated superficial vein of the left limb length discrepancy, lymphedema, soft tissue infection, and pain.
leg and an increased flow velocity. Which of the following is the most likely diagnosis?

B. Elephantiasis
C. Juvenile idiopathic arthritis
D. Klippel-Trenaunay syndrome
E. Sturge-Weber syndrome

86 A 67 year old man has worsening chronic lower limb ischemia. He is scheduled for a A. Great Saphaneous Vein A The great saphenous vein is the most common vein utilized during bypass procedures. Graft choices may https://www.ncbi.nlm.nih.gov/pmc/articles/PMC354964
femoro-popliteal by-pass surgery. Which vein will the surgeon most likely harvest for include vein graft (reversed or in situ), synthetic (polytetrafluoroethylene graft (e.g. Gore-Tex). 9/
the by-pass?
B. Basilic Vein
C. Popliteal Vein
D. Short Saphaneous Vein
E. Cephalic Vein

87 A 72 year old man comes to the clinic complaining of pain of his right foot during the A. Critical Limb Ischemia A Critical Limb Ischemia due to chronic arterial occlusion in the right limb. The lack of pulses indicate it is an https://emedicine.medscape.com/article/761556-overview
night for the last 3 months. He works in construction and mentions that he had arterial problem and it is chronic because there was claudication for over 2 years. Furthermore noctural rest
intermittent calf pains for over 2 years. He has smoked for the last 40 years and drinks pain and ulcerations is a key finding for critical limb ischemia.
beers occasionally. The last time he went to a doctor was 10 years ago. On physical
examination, he has no palpable pulses at the ankle and the popliteal and a small
ulcer on the bottom of his foot. What is the most likely diagnosis?
B. Myopathy
C. Deep Venous Thrombosis
D. Diabetic Neuropathy
E. Trauma from occupation

88 A 40 year old man has been recently diagnosed with hyperlipidemia and A. Daily 1 hour walks as exercise E Quit smoking. Smoking is considered the #1 lifestyle modification to reduce the risk of atherosclerotic events https://www.nhlbi.nih.gov/health-topics/smoking-and-
hypertension. He has a 25 pack year history of smoking and drinks a beer every night. and arterial occlusions. your-heart
He works an office job and finds himself sitting during majority of the day. His BMI is
32 and his weight has not changed much in the last 10 years. He remembers his father
suffering from leg pains and he really wants to avoid that by changing his current
lifestyle. What is the most important lifestyle modification for this patient?

B. Quit his office job


C. Eat less red meat and fried foods
D. A glass of wine every night instead of beer
E. Quit Smoking

89 A 73 year old man comes for a routine health examination. He has well controlled A. Reassurance B Carotid Endarterectomy is indicated for symptomatic carotid artery stenosis > 70%. Bilateral would not be https://www.uptodate.com/contents/management-of-
diabetes and smoking history of 40 pack years but has quit 8 years ago. He reports required as the right carotid is only 50% stenosed and likely asymptomatic. Carotid artery stenting is secondary asymptomatic-carotid-atherosclerotic-disease
weakness in the right arm which occured two weeks ago and lasted two days. On to carotid endarterectomy as there is increased risk of stroke.
examination, systolic bruits are heard over the neck bilaterally. USG is performed
which shows 85% stenosis on the left carotid and 50% stenosis on the right. Whats
the most appropriate next step in management?
B. Left Carotid Endarterectomy
C. Atorvastatin Therapy only
D. Carotid Artery Stenting
E. Bilateral Carotid Endareterctomy

90 A 55 year old woman comes to the outpatient clinic because of the pain and an A. Trendelberg Test C Doppler Ultrasonography is the preferred method for making the diagnosis of chronic venous insufficiency. The https://emedicine.medscape.com/article/1085412-
itching sensation in both her legs for the last year. She works in a clothing retail store Doppler color imaging helps more in mapping the reflux points and planning future surgeries. overview
and mentions that she is on her feet for more than 8 hours a day. She says her feet
feel heavy at the end of the day and she feels relief when she elevates her legs with a
pillow. On examination, you notice mildly distended veins below her right knee. What
is the best investigative tool to help make a diagnosis?
B. Computed Tomography Angiography
C. Doppler Ultrasonography
D. XRay
E. No testing required for diagnosis

91 A 45 year old truck driver shows up to the outpatient complaining of pain in his right A. Ischemia of the right leg E The clinical vignette shows that the patient most likely has DVT. Pulmonary Embolism is a major risk that has https://emedicine.medscape.com/article/1911303-
leg. It started this morning after he came back from a long haul last night. He also to be considered with patient suffering from DVT as untreated PE has a mortality rate of 30%. overview
mentions that he had a difficult time getting his right shoe on and that it still feels
very tight and uncomfortable. On physical examination, there is swelling in his right
leg. What is the most dangerous secondary complication that may occur with this
patient?
B. Ulceration of the foot
C. Apoptosis of limb nerves leading to paralysis
D. Chronic Arterial Occlusion
E. Pulmonary Embolism

92 A 62 year old woman is admitted to emergency department on account of severe leg A. Compartment Syndrome D Virtually all myocytes are irreversibly injured after six hours of ischemia. Flow of blood back into ischemic https://www.ncbi.nlm.nih.gov/pubmed/8003464
pain. Acute limb ischemia is diagnosed and plans for revascularization surgery are tissue leads to further tissue injury and apoptotic components and inflammations markers being circulated
being drawn up. Upon examination, the patient reveals that the pain started 8 hours throughout the body. This ends up paralleling whole body inflammation endangering the life of the patient.
ago. What is the most likely post operation complication that might occur? Amputation would be the best course of treatment.
B. Muscle Atrophy
C. Weight loss
D. Reperfusion Injury
E. Paralysis

93 A 71 year old man arrives to the outpatient clinic because of severe nocturnal right leg A. Amputation A Chronic Lower Limb Ischemia has progressed to critical limb ischemia in this patient. The preferred treatment https://www.jvascsurg.org/article/S0741-5214(19)30321-
pain and gangrene. He was diagnosed with chronic lower limb ischemia 5 years ago choice is amputation because the leg has undergone irreversible damage. Further revascularization procedures 0/pdf
and hyperlipidemia that was not well controlled. He has had two angioplasties in the will not benefit the patient. Such future surgeries may actually impose a severe risk to the patient's health.
last two years and has had recurrent infections in his right limb. The decision is made
to operate on the patient in the attempt to relieve his pain and save his life. What is
the best surgical procedure for this patient?
B. Angioplasty and stenting
C. Endareterctomy
D. Bypass Graft
E. Negative Pressure Wound Therapy

94 A 78 year old caucasian man was admitted to the ER, with sudden pain in the A. Ultrasonography B. US can be used to detect the presence of an AAA, but it has a low sensitivity to detect a retroperitoneal https://www.esvs.org/wp-
abdomen and back. The pain is described as "tearing". On admission his HR is hemorrhage. CTA is the gold standard for all patients with suspected rAAA. It allows to image the patients content/uploads/2018/12/Wanhainen-A-et-al-ESVS-AAA-
140/min, his BP 68/39 mmHg and his RF is 22/min. GCS is 15, ECG shows a sinus vascular anatomy and is crucial in determination whether EVAR or open surgery is required. GL-2019-epublished-041218.pdf p.34 (5.1)
tachycardia without changes in the ST-segment. Abdominal examination reveals a
pulsating mass. Doctors suspect a symptomatic or ruptured AAA and decide to
operate, using EVAR as the method of choice. Considering the findings, which imaging
method is inevitable beforehand?
B. CTA
C. MRI
D. Xray
E. Catheter Angiography

95 A 73 year old man is at the GP for check-up. Incidentally the GP discovered a AAA with A. regular abdominal US to follow the course of the A. With an aneurysm <5cm and no symptoms regular US monitoring is recommended, for aneurysms 4.0-4.9cm https://www.esvs.org/wp-
a diameter of 4,5cm on ambdominal US. The patient certifies he was unaware of this aneurysm over time annual surveillance is safe,drugs to reduce AAA growth rate & cardiovascular risk. Surgical treatment is needed content/uploads/2018/12/Wanhainen-A-et-al-ESVS-AAA-
condition and never experienced pain in the back or abdomen. He did not yet suffer if the aneurysm is >5,5cm or increasing by more than 0,5cm in 6 month. Emergency surgery - in case of GL-2019-epublished-041218.pdf p.16 (chapter 3)
from thromboembolic events. His extremities do not show signs of peripheral artery symptomatic or ruptured AAA.
diseases. He has hypertension, but it is under control with medications. He smokes
occasionally and his cholesterol levels are high. What would you recommend the
patient as the next step?
B. emergency surgery
C. urgent CT to confirm ultrasound diagnosis
D. elective surgery
E. regular CT with contrast to follow the course of the
aneurysm over time

96 A 59 year old man visits the physician for his regular ultrasonic follow-up of his AAA. A. continue to minimize risk factors and come again for D. A growth of 0,5cm in 6 months is an indicator for elective surgical intervention. https://www.esvs.org/wp-
The last years he did not observe any symptoms. He was also really compliant in follow-up in 6 month content/uploads/2018/12/Wanhainen-A-et-al-ESVS-AAA-
minimizing his risk factors and over-all he feels good. The diameter was at a stable GL-2019-epublished-041218.pdf p.18 (recommendation
4cm over the last year (last measured - 6 months ago), this time we observe a 24)
diameter of 5cm in US. What would you recommend to your patient as his surgeon?

B. emergency open repair surgery


C. emergency EVAR surgery
D. elective surgical correction
E. nothing, as long as the patient does not show symptoms

97 A 69 year old man visits the office of his family physician with repeat episodes of A. MRI B. Doppler ultrasound is the next best diagnostic test. It is quick, easy to do and cheap. You can visualize the https://patient.info/doctor/subclavian-steal-
dizziness and weakness of his left arm. He explains that the symptoms usually arise stenosis and the retrograde flow from the vertebral arteries if SSS is suspected. phenomenon#nav-3
when he is doing some kind of activity. The last episode started after he was playing
catch with his grandson. He has a previous medical history of hypertension diagnosed
10 years ago, which he is not taking medication for. BP is 150/90 in his right arm and
110/60 in his left arm. What is the next best diagnostic test?
B. Doppler ultrasound
C. Electromyography
D. Shoulder CT
E. Chest X-Ray

98 A 25 year old Asian woman comes into your office due to repeat problems of A. Subclavian steal syndrome A. Subclavian steal syndrome. Asian women with takayasu arteritis are at an increased risk, up to 36%, for https://patient.info/doctor/subclavian-steal-
maintaining balance and pain in her throwing arm which she notices during or shortly developing SSS. phenomenon#nav-3
after playing softball. It usually gets better after she rests. She has been previously
diagnosed with Takayasu’s Arteritis, for which she is taking appropriate medication.
She says that this dizziness and pain is different from the symptoms she gets from
Takayasu flares. What is the most likely diagnosis?
B. Rotator cuff tear
C. Transient ischemic attack
D. Medication induced side effect
E. Thoracic outlet obstruction

99 During a routine medical checkup a 58 year old man is found to have a difference in A. Routine checkups every 6 months C. In asymptomatic patients, it is best to start with conservative management, such as medical therapy. Every https://patient.info/doctor/subclavian-steal-
upper extremity BP >25 mmHg. He feels completely healthy and has no complaints. patient with SSS should receive some form of treatment because of the increased risk of CVD complications. phenomenon#nav-3
The rest of the physical exam is unremarkable. Doppler ultrasound is performed on
his upper limbs and shows subclavian artery stenosis in his right subclavian artery. The
patient says he feels fine. What is the best initial management for this patient?
B. Bypass surgery
C. Initiate medical therapy (ACE-I, Aspirin, Statin, Beta
Blocker)
D. Stent placement
E. Endarterectomy

100 A 50 year old man is at the doctors office asking about subclavian steal syndrome. He A. Coolness in extremity D. Most patients are in fact asymptomatic. All mentioned symptoms may be present however most patients https://www.ahajournals.org/doi/10.1161/CIRCULATIONA
tells the physician that his father refused treatment after being diagnosed with the present without any symptoms. HA.113.006653
disease and ended up dying from cardiovascular complications. He wants to know
what the most common symptom is of Subclavian steal syndrome?
B. Dizziness
C. Arm claudication
D. Asymptomatic
E. Diplopia

101 A 69-year-old man presents with right arm and hand weakness that resolved within A. Aspirin A. Patient is having a TIA and needs to start antiplatelet therapy due to carotid artery stenosis. Patients with https://www.statpearls.com/kb/viewarticle/18966?utm_s
an one hour. His symptoms started while he was washing the car. His past medical syptomatic cartodid stenosis require surgical intervention when stonsis exceeds 70%. A TIA may present as a ource=pubmed
history is notable for diabetes, anxiety, hypertension, and dyslipidemia. Currently his consequence of other etiologies such as atrial fibllilation. 20-30% of ischemic strokes are caused by carotid
medications include insulin, metformin, and fluoxetine. Examination reveals a right artery stenosis.
carotid bruit. Ultrasound duplex of his carotid arteries demonstrates right and left
carotid stenosis of 55% and 40%, respectively. Which of the following is the best next
step in management?
B. Carotid Artery Stenting
C. Endarterectomy
D. Observation
E. Routine checkups every 6 months

102 A 66-year-old man is evaluated for a carotid bruit detected on routine medical A. Carotid endarterectomy E. The patient is asymptomatic and well controlled using his current medications, regular follow-up and https://www.ncbi.nlm.nih.gov/books/NBK536913/
examination (USG showed left ICA stenosis between 60-80%). He reports no history of cardiovascular assessment is all that is requiered.
previous focal neurologic symptoms or visual loss. He has type 2 diabetes mellitus and
hyperlipidemia treated with metformin, pravastatin, and aspirin. All other physical
examination findings, including those from a neurologic examination, are
unremarkable. Which of the following is the most appropriate next step in
management?
B. Carotid stenting
C. Magnetic resonance angiography of the neck
D. Replacement of aspirin with clopidogrel
E. Control of HTN, lipids, DM

103 A 60-year-old man presents with a visual disturbance that occurred one day ago. He A. strabismus C. Ipsilateral amaurosis fugax is a transient visual disturbance caused by a microembolus in the central retinal https://www.ncbi.nlm.nih.gov/books/NBK536913/
reports that he experienced a “curtain coming down” over his left eye that lasted a artery, known sometimes as visual transient ischemic attack.
few minutes. His past medical history includes long-standing type 2 diabetes mellitus,
hypertension, coronary artery disease, and a 20 year smoking history. On physical
exam, he has bilateral carotid bruits. An ultrasound duplex of his carotids shows
bilateral stenosis with 40% on the right and 75% on the left. What cerebral event has
the patient experienced?
B. blepharospasm
C. amaurosis fugax
D. eyelid ptosis
E. diplopia

104 A 65-year-old female patient with a history of smoking presents to the hospital with A. MRA and CTA D. Patients should always be advised to control their risk factors for atherosclerosis: HTN, smoking, DM, https://www.ahajournals.org/doi/full/10.1161/STROKEAH
mild chest discomfort. She has a history of type 2 diabetes and hypercholesterolemia dyslipidemia. A.115.003390
controlled through metformin and paravastatin. Her bloodwork is unremarkable but
does mention she had a TIA 8 months ago. A duplex carotid ultrasound was
performed showing bilateral carotid narrowing of 50% of the left ICA and 60% of the
right ICA. What is the next step in management?
B. Carotid stenting
C. Carotid endarterectomy
D. Smoking cessation
E. No further treatment or intervention

105 A 69 year old male presents with a one month history of severe cramping leg pain A. Diabetic Neuropathy C. The patient suffers from chronic lower limb ischemia (peripheral artery disease) and is presenting with https://www.uptodate.com/contents/table-of-
which occurs after walking for two blocks and resolves with rest. There is no symptoms of claudication (lower extremity cramping pain on exertion which is relieved by rest). The patient contents/cardiovascular-medicine/peripheral-artery-
associated swelling or erythema of the legs. The patient suffers from T2DM, has risk factors for the disease including diabetes, a long smoking history and hypertension. The fact that he disease
hypertension and has hypercholesterolemia. The patient has had no numbness, has absent pulses further indicates how blood flow is being occluded to the distal extremities of the lower
tingling, weakness, or swelling. Current medications include metformin, and limb. Diabetic neuropathy(A) may present with a "stocking glove" distribution of paresthesia which is not seen
atorvastatin. He has a 40 year pack smoke history. His pulse is 90 bpm, and BP is in this patient. Pseudoclaudication (B) is related to spinal nerve compression leading to position dependent
142/90. The dorsal pedal and posterior tibial pulses on the right and on the left are claudication which is relieved by leaning forward and aggrevated by remaining in an upright position. A DVT(D)
absent. Patellar and Achilles deep tendon reflexes are present bilaterally. Which of presents acutely in patients with lower limb erythema, swelling and pain after prolonged immobilization or in
the following is the most likely cause of this patient condition? hypercoaguable states - our patient has a more chronic presentation. Finally arthirits(E) may be present in a 69
year old patient and present with pain in the lower extremity but the pain would not be "crampy" in nature
and given the risk factors as well as the weak pulse and decreased DTRs the better option would be chronic
lower limb ischemia.
B. Pseudoclaudication (Lumbar Spinal Stenosis)
C. Chronic lower limb ischemia
D. Deep Venous Thrombosis
E. Arthritis
106 Which of the following symptoms is most consistent with high-grade stenosis at the A. Claudication of hip, buttocks, and thighs A Leriche syndrome presents with a classic triad of claudication, impotence and absence of femoral pulses. www.ncbi.nlm.nih.gov/books/NBK538248/
aorto-iliac bifurcation?
B. Abdominal cramps
C. Bleeding
D. Leg ulcers
E. Parasthesia

107 72 year old man presents with complaints of erectile dysfunction, claudication and A. AAA C Leriche syndrome presents with a classic triad of claudication, impotence and absence of femoral pulses. www.ncbi.nlm.nih.gov/books/NBK538248/
absent femoral pulses. What is the diagnosis?
B. Buergers disease
C. Leriche syndrome
D. Deep Venous Thrombosis
E. Spinal stenosis

108 72 year old man presents with complaints of erectile dysfunction, claudication and A. ABI A ABI is the first screening test for chronic lower limb ischemia, followed by angio CT if surgical intervention is www.ncbi.nlm.nih.gov/books/NBK538248/
absent femoral pulses. Which of the following is the next best step? indicated.
B. EKG
C. Doppler USG
D. CT angiogram
E. MRI

109 72 year old man presents with complaints of erectile dysfunction, claudication and A. Axillofemoral bypass C PTA is the intervention of choice in a patient with multiple comorbidities, if the lesion qualifies for www.ncbi.nlm.nih.gov/books/NBK538248/
absent femoral pulses. He has a history of COPD requiering home oxygen. CT with endovascular treatment.
contrast shows bilateral critical stenosis of external illiac arteries. Which of the
following is the most appropriate procedure for this patient?
B. Aortobifemoral bypass
C. PTA + stents
D. Illiac endarterectomy
E. TPA

110 A 55 year-old presents with a 2 month history of claudication of his lower limbs - a A. Ambulatory blood pressure reading B The patient presents with classical symptoms of chronic lower limb ischemia (peripheral artery disease) with https://step2.medbullets.com/cardiovascular/120033/arte
distance of 200m. The patient has a 30-year smoking history (2 packs per day) suffers claudication on exertion, weak peripheral pulses, and a shiny appearance of the lower limbs. Ankle Brachial rial-insufficiency
from hypertension for 20 years and is a diabetic. His BP is 145/90, pulses of the lower index is the best initial test for PAD while doppler ultrasonography can be used to further evaluate the extent
limbs are +1 bilaterally and there is a shiny appearance of the lower limb. Which is the of vascular disease. Ankle brachial index testing involves measuring the blood pressure of the arms and of the
best initial test for the diagnosis of his condition? lower extremities. Values below 1 indicate development of PAD while values <0.4 indicate severe PAD. MRI of
the spine (D) would be useful in patients with lumbar spinal stenosis which present with position dependent
claudication. Ambulatory blood pressure readings (A) can be used in the initial diagnosis of HTN in patients.
Although nocturnal penile tumescence testing (C) would allow us to differentiate between organic and
psychogenic causes of impotence, it is not the best initial test to diagnose PAD. Observation (E) would be of no
benefit.
B. Ankle Brachial Index
C. Nocturnal penile tumescence testing
D. MRI of the spine
E. Observation

111 A 63 year old male presents to the ER with burning pain of the lower extremity at rest A. Heparin D. The patient presents with risk factors and symptoms of chronic limb threatening ischemia probably as a result https://www.uptodate.com/contents/lower-extremity-
(foot). On physical examination you note gangrene, ulceration and hairlessness of the of peripheral artery disease. Our patient has numerous risk factors for PAD including smoking history and surgical-bypass-
lower limb. The patient also admits suffering from progressively worsening poorly controlled diabetes. Hairlessness of the lower limb and impotence are common symptoms of PAD. techniques?search=peripheral%20artery%20disease%20sur
claudication for the past 20 years. He has had poorly controlled diabetes for 20 years However, what leads us to the diagnosis of chronic limb threatening ischemia is the severity of the disease gery&source=search_result&selectedTitle=1~150&usage_t
and is non-compliant with his medications. What intervention would provide the namely the presence of pain at rest, gangrene and ulcerations. Although it is true that medical management ype=default&display_rank=1
most benefit for this patient? such as cilostozal may help patients in earlier stages of the disease presenting with claudication, the question
asks what intervention would provide the most benefit to the patient in this late stage of the disease - which
would be revascularization of the tissue through endovascular surgical means. Surgical bypass would be an
alternative intervention which we can use for our patient to prevent progression which would eventually lead
to amputation (B) Heparin(A) would not be useful in this scenerio nor would aspitin(E) which is an antiplatelet
drug. Observation(C) would be of no benefit to a patient who is in such an advanced stage of the disease.

B. Primary amputation
C. Observation
D. Surgical revascularization
E. Aspirin
112 A 67 year old African American patient presents for an annual check up. His BP is A. Stage I C The Fontaine Scale is a means by which doctors may stage the severity of patients suffering from peripheral https://www.angiologist.com/arterial-disease/fontaine-
145/95 with a temperature of 36.8 degrees C. The patient was diagnosed with artery disease. The scale consists of five stages: Stage 1: asymptomatic(Answer A) Stage 2 a: claudication after classification/
peripheral artery disease 5 years earlier. Following a physical examination of the walking over 200 metres. Stage 2b: claudication after walking less then 200 metres. Stage 3: Pain at rest. Stage
patient, you do not observe any gangrene or ulcerations. You also note weak dorsalis 4(Answer D): Ulcers and gangrene of the limb. Our patient clearly has risk factors associated with PAD
pedis and tibial pulses and thick, shiny skin on lower extremity. The patient admits to (smoking, diabetes, hypertension). As our patient presents with pain at rest she is in the third stage as she has
a 30-year smoking history and poorly controlled diabetes. She complains of pain in pain at rest but does not present with gangrene or ulcers which would place her in stage 4. There is no stage
the lower extremities when walking 100m as well as at night - especially of the foot. 5(answer E) on this scale - the division of stages 2 into - stage 2a and 2b are what make the scale consist of 5
Which stage would you classify this patient according to the Fontaine Scale? stages total.

B. Stage II
C. Stage III
D. Stage IV
E. Stage V

113 A 75 year old man with claudication and rest pain of his left limb lasting for 4 weeks is A. Stage 2 D. Rutherford's Classification: Grade 0, Category 0: asymptomatic https://www.ahajournals.org/doi/pdf/10.1161/CIRCINTER
referred to the vascular surgery outpatient clinic. Upon inspection of the left limb he Grade I, Category 1: mild claudication VENTIONS.115.001946
is noted to have minor tissue loss of the left big toe due to an ulcer. How severe is his Grade I, Category 2: moderate claudication
disease according to the Rutherford classification? Grade I, Category 3: severe claudication
Grade II, Category 4: rest pain
Grade III, Category 5: minor tissue loss; ischemic ulceration not exceeding ulcer of the digits of the foot
Grade IV, Category 6: major tissue loss; severe ischemic ulcers or frank gangrene
B. Stage 3
C. Stage 4
D. Stage 5
E. Stage 6

114 A 62 year old woman is visiting her primary care physician due to calf pain. Once a A. I B. Stage I – Asymptomatic https://www.ahajournals.org/doi/pdf/10.1161/CIRCINTER
week she joins her daughter and her baby granddaughter for a stroll in the park, but VENTIONS.115.001946
has the last couple of times experienced pain about half-way through the 1 km route. Stage I includes patients who are asymptomatic for most of the time, but in whom a careful history may reveal
The pain is relieved by a short rest on a nearby bench, after which she is able to non-specific, subtle symptoms, such as paresthesia. Physical examination may reveal cold extremities, reduced
complete the route and walk home. How severe is her disease according to the peripheral pulse or murmurs in the peripheral arteries.
Fontaine classification?
Stage II – Intermittent claudication. Patients usually have a constant distance at which the pain appears:

Stage IIa – Intermittent claudication after more than 200 m of walking.

Stage IIb – Intermittent claudication after less than 200 m of walking.

Stage III – Rest pain. Rest pain appears especially during the night when the legs are raised up on to the bed,
which diminishes the gravitational effect present by day; also, during the night, the lack of sensory stimuli
allows patients to focus on their legs.

Stage IV – Ischaemic ulcers or gangrene (which may be dry or humid)


B. IIA
C. IIB
D. III
E. IV

115 A 67 year old man is seen by a vascular surgeon due to claudication (walking capacity A. An ABI of 1.1 is within the normal ratio, so there is no C ABI of 1.1 in a symptomatic, diabetic patient is likely a false negative result due to a noncompressible arterial https://www.ncbi.nlm.nih.gov/pubmed/21514102
50m) and rest pain of his left leg. His past medical history includes long standing DM, reason for concern. wall. ABI measurement takes the highest brachial and highest ankle (dorslis pedis artery or posterior tibial
CAD, and renal insufficiency. As a part of the diagnostic work up an Ankle-to-Brachial artery) pressures for each side generally. ABI<0.90 abnormal, rest pain appears at <0.30.
Index is performed, and the patient has an ABI of 1.1. How should this result be
interpreted?
B. An ABI of 1.1 is to be expected in patients with rest pain
and indicates severely reduced blood flow to the limb.

C. An ABI of 1.1 is likely a false negative result in this case,


due to calcified, noncompressible vessles.
D. An ABI of 1.1 is within the normal ratio, however due to
high level of suspicion of PAD, the vascular surgeon should
repeat the test at a later time.
E. An ABI of 1.1 is an indication for emergency surgery.
116 66 y/o man was admitted to the hospital with transient monocular visual loss A) Quit smoking only C Carotid endarterectomy has proved to be beneficial in the prevention of stroke in selected patients. The https://www.ninds.nih.gov/disorders/patient-caregiver-
(amaurosis fugax) and numbness of the right side of his body. Left neck bruits were procedure is indicated in symptomatic patients with carotid-territory transient ischemic attacks or minor education/hope-through-research/stroke-hope-through-
noted. Past medical history includes smoking a pack of cigarettes each day for 30 strokes who have carotid artery stenosis of 70 to 99 percent (Our patient has 90% stenosis). research/questions-answers-carotid-endarterectomy
years, hypertension, and DM. Medications include an ACE inhibitor, a statin, and
repaglinide. Upon diagnostic tests, he is found to have 90% stenosis of the left
internal carotid artery. What would be the best management for this patient?
B) Review current medications and add an adjunctive.
C) Carotid Artery Stenting or carotid Endarterectomy and
lifestyle changes (quit smoking, etc)
D) Carotid Artery Stenting or carotid Endarterectomy only
when the patient reaches 95% stenosis
E) Observation

117 A 21-year old male was admitted with painful swelling of the right arm for the past 3 A) Radial artery B The pathogenesis of effort thrombosis involves extrinsic compression of the subclavian vein between the https://www.ncbi.nlm.nih.gov/pmc/articles/PMC3348760/
weeks. He is a competitive handball player and has been exercising 6 days a week for clavicle and first rib. ?fbclid=IwAR3msRPd167yFu7KA8X3GSXn__dT1M4UkAedU
the past few months. A doppler ultrasound would most likely show a thrombosis in Lo5z6ygFbT270ItnPgBJMk
which vessel?
B) Subclavian vein
C) Jugular vein
D) Radial vein
E) Subclavian artery

118 A 30-year old boxer was admitted with painful swelling of the right arm for the past 2 A) Female to male ratio is 2:1 C Paget-Schroetter syndrome is twice as common in males as in females https://www.sciencedirect.com/science/article/pii/S07415
weeks after vigorous exercise. Edema and dilated superficial veins were noted in the 2140902518X
right upper limb. Venous angiography of the right upper limb confirmed the diagnosis
of Paget-Schroetter syndrome. What is the correct statement about this syndrome?

B) Male to female ratio is 1:1


C) Male to female ratio is 2:1
D) Male to female ratio is 3:1
E) Female to male ratio is 3:1

119 A 25-year old body builder presents with a blue, swollen, heavy and painful arm. He A) Varicose veins B Post-thrombotic syndrome is a common complication of deep vein thrombosis if left untreated. https://www.ncbi.nlm.nih.gov/pmc/articles/PMC3348760/
has a vigorous exercise program and is following a strict diet. Physical examination ?fbclid=IwAR3msRPd167yFu7KA8X3GSXn__dT1M4UkAedU
reveals edema and a Doppler ultrasound of the right upper limb shows a thrombosis Lo5z6ygFbT270ItnPgBJMk
of the right subclavian vein. What is a common complication of this syndrome if not
treated appropriately?
B) Post-thrombotic syndrome
C) Hematoma
D) Cellulitis
E) Arthritis

120 A 23-year old male was admitted with a painful swelling of the right arm for more A) CT D Doppler ultrasonography is the preferred initial test for the diagnosis of Paget-Schroetter syndrome. https://www.ncbi.nlm.nih.gov/pubmed/7824774
than 3 weeks. He had been exercising regularly, usually heavy weight lifting for 4
months. On physical examination, edema and dilated superficial veins were noted in
the right upper limb. What is the best next diagnostic step?
B) MRI
C) X-ray
D) Doppler ultrasonography
E) Arterial bypass graft assessment

121 A 44y old woman comes to your office with an ulcer at the dorsum of the left heel. A Hypertension C Smoking is by far the greatest risk factor for developing peripheral artery disease (more than 4x chance of https://www.heart.org/en/health-topics/peripheral-artery-
The patient does not remember when the ulcer first appeared but that it has been getting it). This patient presents with many risk factors and all need to be controlled and treated properly. The disease/understand-your-risk-for-pad
there for "quite some time." The patient is diabetic and takes ACEi for HTN which has patient should stop smoking immediatly, exercise to reduce weight and monitor blood glucose levels, as well
moderate results. History reveals that she has smoked one pack of cigarettes for 15 as the blood pressure closely. All this to reduce further development of artherosclerotic plaques and damaging
years and has a BMI of 33. Further evaluation reveals that the patient is suffering other vessels.
from peripheral artery disease and that the left leg has been hypoperfused for some
time. The diagnosis of chronic lower leg ischemia is made. What is the greatest risk
factor for developing peripheral artery disease?
B Obesity
C Smoking
D Genetic predisposition
E Diabetes
122 A 56y old man comes in to ER with acute limb ischemia of the left limb. The vascular A 1 B The given findings are characteristic for Rutherford class IIa. Minimal sensory deficit (toes), no motor https://teachmesurgery.com/vascular/peripheral/acute-
surgeon asks the resident to assess the patient's lesion according to the Rutherford dysfunction, an audible venous and inaudible arterial Doppler. Class I has no sensory deficit and an audible ischaemia/
classification. The resident finds sensory loss on the toes, good muscle tone and arterial Doppler. Class IIb has more extensive sensory loss with mild to moderate motor dysfunction. There will
motor function. On Doppler ultrasonography, the arterial component is inaudible also be some pain at rest. The patient will have the same Doppler findings as with IIa. Grade III has profound
while the venous can be heard. Which stage of the Rutherford classification is correct sensory loss, muscle paralysis and inaudible venous and arterial Doppler.
for this patient's condition?
B 2a
C 2B
D3
E4

123 A 67y old man comes to the ER following intense leg pain. After careful physical A Administer heparin C The patient is suffering from compartment syndrome. An emergency fasciotomy is required as soon as possible https://www.nhs.uk/conditions/compartment-syndrome/
examination, the doctor diagnoses acute lower leg ischemia and undergoes to not cause further damage of the surrounding tissue in the leg. It is often caused by reperfusion injury after
thrombectomy to revascularize the leg. The next day after the operation, the patient revascularization of an limb which tends to cause increased capillary membrane permeability. The doctor
experiences tightness, pain and numbness in the affected limb. What is the next step makes an incision of the skin and fascia of the surounding muscle and relieves the pressure. The wound will
in management? then often be closed a few days later.
B Doppler ultrasound
C Emergent fasciotomy
D X- ray of the leg
E CT- angiography

124 A 27-year-old women in her 10th week of pregnancy was referred to the hospital. On A. Leg Edema B Homan’s sign is for deep venous thrombosis, combined with other finding and positive for duplex ultrasound https://www.ncbi.nlm.nih.gov/pmc/articles/PMC5778511/
examination, her blood pressure, heart rate and respiratory rate were 110/80 mmHg, which confirms the diagnosis
85 per minutes and 14 per minutes, respectively. Deficiency of antithrombin with high
coagulation factors were observed and she is positive for Homan’s sign. What is most
likely the diagnosis?
B. Deep venous thrombosis
C. Superficial thrombophlebitis
D. Pulmonary Embolism
E. Normal symptoms for Pregnancy

125 A 26-year-old women is found to be on her 4th week of pregnancy pregnant when she A. Wait for the lab results to initiate treatment B Even though the lab result is still in evaluation, treatment with LMWH should be initiated for prevention of the https://www.ncbi.nlm.nih.gov/pmc/articles/PMC5778511/
came to the hospital due to pain and swelling of the left lower limb. On examination, symptoms of pulmonary embolism or further worsening. The duplex ultrasound is positive which is enough to
the affected limb is swollen, warm and shows redness. A blood sample was sent to confirm the DVT diagnosis. LMWH is preferred over warfarin. Warfarin is contraindicated in pregnancy.
the lab for D-dimer evaluation. The patient’s duplex ultrasound is positive for DVT of
the affected limb. What is the next step of management?
B. Start anticoagulant therapy with LMWH, while waiting
for the lab results.
C. Start anticoagulant therapy with warfarin, while waiting
for the lab results.
D. Reassure that the symptoms will improve with an
antibiotic
E. Need further investigation for diagnosis

126 A 35-year-old women in her 15th week of pregnancy referred to the hospital because A. Duplex Ultrasound D A bronchial challenge test is not diagnostic for deep vein thrombosis or pulmonary embolism, instead it is used https://www.ncbi.nlm.nih.gov/pmc/articles/PMC5778511/
of dyspnea with swelling and pain of the lower limbs. Examination and Lab evaluation for the diagnosis of asthma.
have been performed and confirm diagnosis of deep vein thrombosis with pulmonary
embolism. Which of the following is NOT a diagnostic test of choice for DVT+PE?

B. D-Dimer testing
C. Spiral CT
D. Bronchial challenge test
E. Pulmonary Angiogram

127 A 22-year old women in her 13th week of her pregnancy referred to the hospital A. Fleischner sign C Homan’s sign - a sign for deep venous thrombosis on physical examination - a dorsiflexon sign test is a physical http://www.jadweb.org/article.asp?issn=2221-
because of Hemoptysis and shortness of breath. Physical examination found swelling examination procedure that is used to test for Deep Vein Thrombosis (DVT). A positive Homan’s sign in the 6189;year=2018;volume=7;issue=3;spage=99;epage=102;a
and stasis of both lower limbs. Laboratory finding, D-dimers and BNP values were presence of other clinical signs may be a quick indicator of DVT. Clinical evaluation alone cannot be relied on ulast=Shawn
elevated with deficiency of protein S activity. Chest roentgenogram revealed mild for patient management, but when carefully performed, it remains useful in determining the need for
hilar enlargement which can be caused by pulmonary hypertension or pulmonary additional testing (like D-dimer test, ultrasonography, multidetector helical computed axial tomography (CT),
embolism. Which of the following is not a sign on chest roentgenogram that could be and pulmonary angiography).
useful to indicate pulmonary embolism?
B. Westermark sign
C. Homan’s sign
D. Hampton’s sign
E. Palla’s sign
128 A 75 year old man comes to the ER due to sudden onset of pain in the distal part of A. CT – Angiography A CT arteriogram can provide more information regarding the anatomical location of the occlusion and can help https://teachmesurgery.com/vascular/peripheral/acute-
his right calf. His right leg is pale, cold and pulseless and he can't feel his toes. What is decide the operative approach as the patient is uffering from acute limb ischemia. ischaemia/
the gold-standard investigation:
B. Bedside doppler ultrasound
C. X-ray
D. Digital subtraction angiography
E. Ankle-Brachial index

129 55 year old man with acute limb ischemia undergoes embolectomy. What post- A. Myocardial infarction E An important complication of acute limb ischaemia is reperfusion injury; sudden increase in capillary https://teachmesurgery.com/vascular/peripheral/acute-
operative complication is the man especially at risk of due to the revascularization? permeability can result in compartment syndrome. Most commonly it requires 4-compartment ischaemia/
(anterior/lateral/superficial and deep posterior) fasciotomy.
B. Urinary retention
C. Seroma
D. Kidney Failure
E. Compartment syndrome

130 A 32 year old drunk male comes in with massive bleeding from a laceration in the left A. Securing patent airways C This patient presents with hemorrhagic shock. And in this case control of the hemorrhage takes precedent over https://www.ems1.com/trauma-assessment/articles/ems-
upper extremity. He is unconcious and his affected extremity is pale and has a the ABCs. trauma-care-abcs-vs-march-vdnYK1J4Pq5cZbnV/
capillary refill time of 11 seconds, in his contralateral extremity the capillary refill is 6
seconds. He has a blood pressure of 85/40 and heart rate of 135 and an oxygen
saturation of 98%. You are the vascular surgeon in the ER and you are with the
trauma team, what is the first priority in the treatment of this patient?
B. Intubation
C.Controlling hemorrhage
D. Check breating rate
E. Administering a vasopressor

131 You are in the OR operating on a 25 year old patient with a laceration of the brachial A. Radial nerve C The median nerve is in very close proximty to the brachial artery and one has to take great care to not injure it https://teachmeanatomy.info/upper-limb/nerves/median-
artery. You are trying to make an end-to-end anastomosis 5 cm above the elbow to during surgery. nerve/
secure proper perfusion to the distal upper extremity. Which structure do you have to
be especially careful not to injure
B. Axillary nerve
C. Median nerve
D. Brachial plexus
E. Cephalic vein

132 A 71 year old patient comes into the vascular surgery department with a referral from A. Ascending aorta B The most common form of an aortic aneurism is in the abdominal aorta. The most common location of a AAA https://surgery.ucsf.edu/conditions--
his GP. His GP suspects an aortic aneurism due to the patients risk factors and is in the infrarenal segment. procedures/abdominal-aortic-aneurysm.aspx
symptoms. A USG confirms the presence of a 4.5 cm aortic aneurism, which location
is the most likely location of the patients aneurism?
B. Infrarenal
C. Bifurcation of aorta
D. Aortic arch
E. Suprarenal

133 A 69 year old lifetime smoker is admitted to the vascular surgery department with a A. The development of back and flank pain A Any development of symptoms excluding all other possible causes is an indication for emergency repair of a https://www.uptodate.com/contents/management-of-
5.1 cm abdominal aortic aneurism. At the moment he has no symptoms and is AAA. asymptomatic-abdominal-aortic-
wondering why he is in the hospital. The patient asks you (the vascular surgeon on aneurysm?search=indication%20for%20aaa%20surgery&so
call) why he is not just operated on to get it over with. You answer that there is no urce=search_result&selectedTitle=1~150&usage_type=defa
indications for surgery at the moment. However, which of the below alternatives ult&display_rank=1#H786575
would be an indication for surgery?
B. No reduction in aneurism size in 3 month control
C. An increase in aneurism size from 5,1 to 5,4 in 18
months
D. The patient being diagnosed with colon cancer with
mets to peritoneum
E. Failure to keep BP below 140/90

134 A patient presents with a palpable pulsatile mass in the right groin two days after a A. DVT E Pseudo aneurysms as complications are a major source of morbidity after cardiac catheterization. Their https://www.ncbi.nlm.nih.gov/books/NBK542244/
coronary intervention procedure. What is the most probable diagnosis? incidence varies in the literature due to different definitions, methods of interrogation and presence of certain
complications. Post-catheterization lesions occur in approximately 0.05% of treated patients after diagnostic
catheterization and up to 1.2% after more complex procedures.
B. Seroma
C. Abcess
D. Postraumatic hematoma
E. Iatrogenic pseudonaeurysm of the right femoral artery
135 What is the investigative study of choice for an uncomplicated iatrogenic femoral A. CT angiogram D In straight forward cases of femoral pseudoaneurysms, a duplex USG is considered the gold standard. A CT https://www.ncbi.nlm.nih.gov/books/NBK542244/
pseudoaneurysm? angio might be considered if adjacent tissue is of a certain interest and surgery is indicated.
b. MRI
c. Clinical diagnosis
d. Duplex USG
e. Angiography

136 What is the treatment of choice for an uncomplicated femoral pseudoaneurysm >4 A. observation B In a femoral artery pseudoaneurysm (FAP) >3 cm in diameter, USG-guided thrombin injection is considered the https://www.ncbi.nlm.nih.gov/books/NBK542244/
cm in diameter in a patient receiving antiplatelet dual therapy? treatment of choice. Observation is reserved for patients with a FAP <3 cm that may spontaneously regress.
Surgery is an option if the thrombin injection fails to alleviate the FAP.
B. USG-guided thrombin injection
C. surgery
D. compression bandages
E. withdrawel of dual antiplatelet therapy

137 What is the most feared complication of a ruptured femoral pseudoaneurysm? A. compartment syndrome B A femoral artery pseudoaneurysm can rupture into the retroperitoneal space causing significant bleeding that https://www.ncbi.nlm.nih.gov/books/NBK542244/
may not be immediately obvious and in worst-case scenario can cause death.
B. retroperitoneal hemorrhage
C. distal embolization
D. DVT
E. acute lower limb ischemia

138 What is the most common cause of a femoral artery pseudoaneurysm? A. spontaneous D The number one cause of femoral artery pseudoaneurysm is iatrogenic. Interventional procedures like https://www.ncbi.nlm.nih.gov/books/NBK542244/
percutaneous transluminal coronary angioplasty is a common etiology. Complications of pseudoaneurysms
include distal embolization, manifestations due to mass effect, and rupture leading to catastrophic bleeding.

B. atherosclerotic plaque rupture


C. trauma
D. iatrogenic
E. longstanding history of uncontrolled hypertension

139 Femoral artery pseudoaneurysm is a so-called false aneurysm. Why is it not A. contains no layers of vessel wall A FAP is considered a false aneurysm since it contains no layers of vessel wall. https://www.ncbi.nlm.nih.gov/books/NBK542244/
considered a true aneurysm?
B. has no pathological weak spots in its wall
C. does not form thrombus, and has no potential of
embolization
D. it is iatrogenic
E. It is of a lesser diameter compared to that of true
aneurysm

140 In a 65-year-old male with stable angina, a prominent aortic pulsation in the A. Aortography followed by annual CT scans. D The most accepted method for monitoring, because of its cost-effectiveness, is the ultrasound scan that should https://medicina.grupocto.es
abdomen is discovered during a systematic physical examination. His doctor orders an be performed every 6 months to detect if there is a large growth (that increases more than 0.5 cm in 6
abdominal CT scan that demonstrates a 3.9 cm diameter aortic aneurysm in the months).
infrarenal location. The most recommended course of action in this case is:
B. A careful assessment of cardiac status and elective
aneurysm repair if the risk is acceptable.
C. Induced aneurysm thrombosis and extra-anatomical
bypass.
D. Serial ultrasound examinations to assess the annual
increase in aneurysm size.
E. No control is necessary.

141 A 50-year-old patient with clinical suspicion of acute aortic dissection type A (Stanford A. CT- angio A. The definitive diagnosis of aortic dissection is made by CT-angiography. Aortography requires the introduction https://medicina.grupocto.es/
Classification) presents in unstable condition. Indicate the relevant diagnostic study to of a catheter into the aorta to selectively inject contrast at that level, which is contraindicated because of the
confirm the diagnosis and schedule surgery. risk of traumatic rupture if there is a separation of the intima-media.
B. Transesophageal ultrasound
C. Transthoracic ultrasound
D. Aortography
E. Chest X-ray

142 What is the most common complication of abdominal aortic aneurysms larger than 6 A. Occlusion of mesenteric arteries E. The risk of rupture of an aneurysm increases with size. The risk of rupture in aneurysms with a diameter less https://medicina.grupocto.es
cm in diameter? than 5 cm is 1-2% at 5 years, while those with a diameter greater than 5 cm reach 20-40%. All answer choices
are possible.
B. Distal embolization.
C. Ureteral compression with secondary hydronephrosis
D. Acute lower limb ischemia
E. Rupture
143 Acute Stanford Type B Thoracic Aortic Dissection is characterized by the following A. Dissection affecting the root of the aorta E. Acute Stanford Type B Thoracic Aortic Dissection is characterized by involvement of the descending aorta, https://medicina.grupocto.es/
anatomical findings: distal to the left subclavian artery outlet By definition, the ascending aorta cannot be affected and therefore
the entire aorta cannot be affected.
B. Dissection affecting the aortic root and aortic valve, but
preserving the rest of the ascending aorta
C. The dissection only affects the aortic arch
D. Dissection involving the entire ascending aorta
E. Dissection of the descending aorta distal to the left
subclavian artery

144 The most recommended therapeutic option at this time for large abdominal aortic A.Aneurysm resection and conventional surgical graft. C. At present, treatment of aneurysms is considered surgical if the risk of rupture is high. As an alternative to https://medicina.grupocto.es
aneurysms in high surgical risk patients is? conventional surgery the treatment of AAAs is with the reconstruction of the vessel via endovascular.
B.Aneurysm growth monitoring
C.Endovascular treatment with stentgraft implantation
D.Induction of sac thrombosis and axillary-bifemoral by-
pass
E. Surgery scheduled in two weeks.

145 59-year-old smoker with a history of high blood pressure presents to the hospital. He A. Acute coronary syndrome C. In a middle-aged male with hypertension, when faced with a clinical case of severe pain that is dislocated and https://medicina.grupocto.es/
went to the emergency department reporting very intense chest pain of sudden onset accompanied by neurological involvement or alterations in peripheral pulses, we must always suspect first that
about 4 hours ago. The location of pain shifted to the interscapular region. On the patient is suffering from an acute aortic dissection.
examination he presented paleness, profuse sweating, TA 190/104 mmHg, HR 108
bpm and very low radial pulses; a diastolic murmur was heard on the left sternal
border. What is the most likely diagnosis?
B. Pulmonary thromboembolism
C. Acute aortic dissection
D. Coronary vasospasm
E. Cardiac infarction

146 A 60-year-old man goes to the emergency department reporting intense, tearing, 1- A. Sublingual nitroglycerin up to 3 times, measure cardiac D. In the case of chest pain of the characteristics described (anterior and/or interscapular, intense, tearing) in a https://medicina.grupocto.es/
hour-long anterior and interscapular chest pain. The BP is 170/110 mmHG in the right enzymes and admit the patient patient over 50 years of age where cardiac pathology is initially ruled out by presenting a normal ECG, a
arm and 110/50 mmHG in the left arm. The ECG shows sinus rhythm with left vascular pathology of the intrathoracic aorta must be suspected, especially if it is associated with a picture of
ventricular hypertrophy. Chest x-ray shows no data of interest. Which of the following hypertension. The pain described is quite characteristic of an aortic dissection and if it is suspected,
is the most appropriate initial intervention? progression should be avoided as much as possible. To this end, intravenous beta-blocker (propanolol) is
administered to reduce cardiac contractility, and nitroprusside to lower blood pressure. Once the patient is
stabilized, confirmation of the vascular pathology should be attempted, with the thoracic CT being considered
the complementary diagnostic method that provides the most information.
B. Administer tissue plasminogen activator intravenously
and admit the patient to the coronary unit
C. Intravenous bolus heparin and continue continuous
infusion, perform pulmonary ventilation/perfusion scan
and admit the patient
D. Intravenous nitroprusside to maintain a <110 systolic BP,
intravenous propanolol to maintain a <60/min heart rate
and perform a chest CT with contrast.
E. Do not make treatment decision until cardiac enzyme
results are obtained and a transesophageal ultrasound is
performed

147 An 80-year-old male patient reports having very intense lower back pain, of sudden A. The most likely diagnosis is a colon neoplasm D. An AAA should be considered because they are more frequent in older men. The existence of a pulsatile mass https://medicina.grupocto.es
onset, at rest and without modification with movements or lumbar palpation. During on palpation makes us suspect the presence of an abdominal aneurysm and hypotension would almost
the physical examination, arterial hypotension and the existence of a pulsatile certainly indicate a rupture of the same. When suspecting a ruptured (complicated) abdominal aneurysm an
abdominal mass stand out. Which of the following statements are true in relation to abdominal CT should always be performed to confirm it, since we are facing a surgical emergency because of
the diagnosis and treatment of the patient? the risk of death suffered by the patient if not treated.
B. The clinical picture suggests aortic dissection and an
aortography should be done immediately.
C. The abdominal mass suggests an AAA but does not
explain the patient's back pain.
D. An immediate abdominal CT scan should be performed
because of the probable existence of a complicated AAA
and urgent surgical evaluation.
E. Evaluation of pharmacological treatment.
148 A 64 year old male with a history of mild hypertension, well-controlled diabetes, a) hypertension B Popliteal artery aneurysm is a rare condition, but it's the most common peripheral aneurysm. Popliteal artery https://www.ncbi.nlm.nih.gov/books/NBK430863/
hypercholesterolemia, abdominal aortic aneurysm repaired with an open surgery 7 aneurysm is associated with abdominal aortic aneurysm in 40-50% of the cases, so looking at the patient’s
years ago (aorto-aortic graft) and knee arthroplasty 3 years ago presents to the history his current complaint is very much likely to be associated with a previous AAA.
vascular surgery department with lower limb discomfort, mild pain and blue
discoloration of the toes. The patient recalls pain in both legs when walking for longer
than 10 minutes. On physical examination there’s a pulsatile mass in popliteal fossa
bilaterally, normal pedal pulses and mild edema. Bilateral popliteal aneurysm is
suspected. What is the strongest predicting factor supporting such diagnosis?

b) previous AAA
c) previous knee arthroplasty
d) age
e) claudication

149 A 78 year old male with a history of uncontrolled hypertension, diabetes and a) recurrence of AAA C Patient presenting symptoms of distal feet ischemia and with a history of AAA is most likely to suffer from https://www.uptodate.com/contents/popliteal-artery-
abdominal aortic aneurysm repaired with an open surgery 15 years ago (aorto-aortic popliteal artery aneurysm bilaterally, which in 40-50% of cases is associated with AAA. Microthrombi can form aneurysm
graft) presents to the vascular surgery department with increasing pain and in the aneurysm due to turbulent flow and endothelial injury and can travel distally lodging in smaller arteries
discomfort in the left leg and blue discoloration of the toes. Embolization of distal causing blue-toe syndrome presenting as blue discoloration of the toes.
arteries is suspected as a cause of blue toe syndrome presented in this patient. What
is the most probable diagnosis?
b) compartment syndrome
c) bilateral popliteal artery aneurysm
d) Buerger’s disease
e) popliteal entrapment syndrome

150 A 67 year old male presents to the emergency department with sudden severe left leg a) left atrium B This patient most probably suffers from acute limb ischemia. Most probable cause of a thrombsis in the case of https://www.ncbi.nlm.nih.gov/books/NBK541115/
pain and numbness for the last 1,5 hour. The patient has a history of abdominal aortic this patient is the popliteal artery aneurysm, having in mind the history of abdominal aortic aneurysm and a
aneurysm treated with end-vascular stent graft, he was recently diagnosed with mass in the popliteal fossa. In this case the mass is not pulsating indicating a critical thrombosis of the
hypertension and underwent a thorough cardiological examination. On physical popliteal artery.
examination the left leg is cold and pale and a mass is found in the popliteal fossa.
Pedal pulse is absent and it is present in the unaffected limb. Duplex ultrasound is
performed and thrombosis of popliteal artery is diagnosed. What is the most probable
source of the thrombus causing the acute limb ischemia?

b) popliteal artery aneurysm


c) femoral vein thrombosis
d) femoral artery aneurysm
e) iliac artery

151 A 48 year old male with uncontrolled hypertension, diabetes and a) superficial femoral vein bypass B The patient has symptomatic popliteal artery aneurysms with diameter exceeding 2 cm, which are both an https://www.uptodate.com/contents/surgical-and-
hypercholesterolemia complains of mild leg pain at rest and more severe while indiction for a repair. The gold standard treatment for popliteal artery aneurysm is saphenous vein bypass. endovascular-repair-of-popliteal-artery-
walking for more than 10 minutes, blue discoloration of the toes and mild tingling aneurysm?search=popliteal%20artery%20aneurysm&sourc
sensation. Duplex ultrasound is performed and bilateral popliteal artery aneurysm is e=search_result&selectedTitle=2~30&usage_type=default&
diagnosed: 2,4 cm in diameter on the left side and 2,1 cm in diameter on the right display_rank=2
side. What is the best form of management?
b) saphenous vein bypass
c) prosthetic bypass
d) duplex ultrasound surveillance every 6 months
e) pharmacotherapy of hypertension, diabetes and
hypercholesterolemia

152 A 50 year old male come to the office complaining of right foot rest pain during the A) Chronic Lower Limb Ischemia A This patient with a history of peripheral artery disease as well as a previous MI placing himself at an increased https://www.ncbi.nlm.nih.gov/books/NBK538248/
nighttime for the past two months. This patient has a prior history of a previous MI, risk for lower limb ischemia. Due to the fact that the patients lower leg pain occurs during the night and for
hypertension, DVT and peripheral artery disease. On further inspection the patients longer than two weeks this points to the only acceptable diagnosis at the time to be Chronic Lower Limb
affected leg is more pale than the other. Which diagnosis can explain this patients leg ischemia, most likely in its critical stage.
pain as well as discoloration?
B) Restless Leg Sydrome
C) Spinal Stenosis
D) Acute Lower Limb Ischemia
E) Herniated Disc
153 A 52 year old obese male who is a 30 year pack a day smoker presents with a history A)Thromboembolism C This patients presentation is most in line with the diagnosis of Leriche syndrome as this is defined as an https://www.ncbi.nlm.nih.gov/books/NBK538248/
of pain and cramping in his legs and buttocks especially when walking or exercising. aortoiliac occlusive disease that is common in elderly smokers and can present with pale legs, erectile
This patient also compains of erectile dysfunction. Physical examination of this dysfunction, and pain on exertion.
patient shows pale and cold legs. Ultrasound of the patients abdominal aorta just
above the site of its bifurcation reveals a artherosclerotic occlusion. What is this
patient’s diagnosis?
B)Spinal Stenosis
C)Leriche Syndrome
D)Arterial Dissection
E)Sciatica

154 A 19 year old physically active male presents to the emergency department with A)Upper arm contusion C This patient is experiencing an effort induced thrombosis known as paget-Schroetter syndrome. Characteristic https://www.ncbi.nlm.nih.gov/pmc/articles/PMC2967689/
painful swelling of his right arm for the past two weeks. The patient practices sports in young athletes this sydrome is the formation of a thrombos in absence of secondary causes.
daily and lifts weights as well. On physical examination the physician noticed arm
edema along with superficial veins in the upper limb. Patient has no
lymphadenopathy, no history of coagulation pathology and a BP of 120/80. What is
this patients diagnosis?
B) Allergic Reaction
C) Paget-Schroetter Syndrome
D) Thoracic Outlet Syndrome
E) Bony Tumore

155 An 22 year old patient presents with a persistent deep ache in her leg along with A)Compartment Syndrome A Compartment syndrome is a painful condition caused by the increase interstitial pressure (intracompart- https://www.ncbi.nlm.nih.gov/pmc/articles/PMC4071472/
swelling, tightening, and bruising following a blunt force trauma injury one week ago. mental pressure – ICP) within a closed osteofascial compartment which impair local circulation. It occurs most
The patient states that the pain has continued to worsen over time, especially with often in the legs, but it can affects also the arms, hands, feet, and buttocks. It usually develops after a severe
physical activity and gets better on rest. All nerve functions are preserved. What is the injury such as fractures or crush injury, but it can also occurs after a relatively minor injury and it may be
most likely diagnosis ? iatrogenic.
B)DVT
C)Venous Insufficiency
D)Cellulitis
E)Tendonitis

156 A 25-year-old man comes to the emergency department complaining of left arm pain a. Superficial Thrombophlebitis C The most likely diagnosis is Paget-Schroetter Syndrome, an upper extremity deep vein thrombosis. Commonly https://www.uptodate.com/contents/primary-
which began after he was lifting weights. On physical exam his left arm is swollen and occurs in young male who exercise often. Roos test is a diagnostic tool used in the identification of Thoracic spontaneous-upper-extremity-deep-vein-
red. Roos test is positive. What is most likely diagnosis? Outlet Syndrome (TOS). It is also known as the “elevated arm stress test” or "EAST". thrombosis?source=history_widget#H25075587
b. Budd-Chiari Syndrome
c. Paget-Schroetter Syndrome
d. Burger Disease
e. Lymphedema

157 A 72-year-old man is hospitalized for an elective aortic aneurysm repair, 5 days after a. Warfarin D The patient has most likely develop HIT as a result of Heparin anti-coagulation. Heparin-induced https://www.uptodate.com/contents/clinical-presentation-
the scheduled surgery he has ecchymoses all over his trunk and lower limbs. Platlet thrombocytopenia (HIT) is a potentially devastating immune mediated adverse drug reaction caused by the and-diagnosis-of-heparin-induced-
count at admission was 175 000 , Platlet count today was 45 000. What is the most emergence of antibodies that activate platelets in the presence of heparin. Despite thrombocytopenia, thrombocytopenia?search=HIT&source=search_result&sele
likely offending agent? bleeding is rare; rather, HIT is strongly associated with thromboembolic complications involving both the ctedTitle=1~150&usage_type=default&display_rank=1
arterial and venous systems.
b. Fondaparinux
c. Rivaroxaban
d. Heparin
e. Aspirin

158 A 71-year-old man is discoved to have a AAA on USG imaging. The diameter of the a. council the patient to modify risk factors of AAA A. The most important modifiable risk factor for AAA is smoking, counceling patients to quit smoking will have https://www.uptodate.com/contents/management-of-
aneurysm is 4 cm. What is the most important next step? the greatest effect at this point. asymptomatic-abdominal-aortic-
aneurysm?search=abdominal%20aortic%20aneurysm&sou
rce=search_result&selectedTitle=1~150&usage_type=defau
lt&display_rank=1#H1258861470
b. prescribe a beta-blocker
c. perform emergent surgery
d. perform elective surgery
e. observation with follow-up in 2 years

159 A 59 year old male undergoes a popliteal bypass procedure (PTFE graft) for an a. Aspirin + Warfarin E Following any bypass or stenting procedure all patients should be begun on DAPT as long as they dont have a https://www.uptodate.com/contents/surgical-and-
aneurysm on his left popliteal artery. After the operation, he is placed on chronic high bleeding risk. Associated with better long term outcomes. endovascular-repair-of-popliteal-artery-
antiplatlet therapy. What is the best regimen? aneurysm?search=popletial%20stenting&source=search_re
sult&selectedTitle=1~150&usage_type=default&display_ra
nk=1#H1231031607
b. Warfarin + Clopidogrel
c. Heparin + Clopidogrel
d. Tecagralor + Warfarin
e. Aspirin + Clopidorel

160 A 64 year old man with a history of a triple coronary artery bypass 2 years ago, A. Warfarin C Lifelong low-dose aspirin should be part of best medical treatment to reduce the risk of stroke and other CV https://www.esvs.org/wp-content/uploads/2015/12/ESC-
presents with peripheral arterial occlusive disease. His only medication is a thiazide events, as these patients are also at twice the risk of MI. Antiplatelet monotherapy is recommended. ESVS-PAD-2017.pdf
diuretic. Which of the following medications would be most appropriate in the Clopidogrel (75 mg/day) is an alternative in patients with aspirin intolerance.
medical management of his atherosclerosis?
B. Low-dose heparin
C. Aspirin
D. Clopidogrel
E. Corticosteroids

You might also like